Sei sulla pagina 1di 116

Short Notes and

Short Cases in
ENT
Short Notes and
Short Cases in
ENT
UN Panda
MD
Senior Physician
New Delhi

JAYPEE BROTHERS
MEDICAL PUBLISHERS (P) LTD
New Delhi
Published by
Jitendar P Vij
Jaypee Brothers Medical Publishers (P) Ltd
EMCA House, 23/23B Ansari Road, Daryaganj
New Delhi 110 002, India
Phones: 23272143, 23272703, 23282021, 23245672, 23245683
Fax: 011-23276490 e-mail: jpmedpub@del2.vsnl.net.in
Visit our website: http://www.jpbros.20m.com

Branches
• 202 Batavia Chambers, 8 Kumara Kruppa Road, Kumara Park East,
Bangalore 560 001, Phones: 2285971, 2382956 Tele Fax: 2281761
e-mail: jaypeebc@bgl.vsnl.net.in
• 282 IIIrd Floor, Khaleel Shirazi Estate, Fountain Plaza
Pantheon Road, Chennai 600 008, Phone: 28262665 Fax: 28262331
e-mail: jpmedpub@md3.vsnl.net.in
• 4-2-1067/1-3, Ist Floor, Balaji Building, Ramkote
Cross Road, Hyderabad 500 095, Phones: 55610020, 24758498
Fax: 24758499 e-mail: jpmedpub@rediffmail.com
• 1A Indian Mirror Street, Wellington Square
Kolkata 700 013, Phone: 22451926 Fax: 22456075
e-mail: jpbcal@cal.vsnl.net.in
• 106 Amit Industrial Estate, 61 Dr SS Rao Road, Near MGM Hospital
Parel, Mumbai 400 012, Phones: 24124863, 24104532 Fax: 24160828
e-mail: jpmedpub@bom7.vsnl.net.in

Short Notes and Short Cases in ENT


© 2003, UN Panda

All rights reserved. No part of this publication should be reproduced,


stored in a retrieval system, or transmitted in any form or by any
means: electronic, mechanical, photocopying, recording, or otherwise,
without the prior written permission of the author and the publisher.
This book has been published in good faith that the material pro-
vided by author is original. Every effort is made to ensure accuracy
of material, but the publisher, printer and author will not be held
responsible for any inadvertent error(s). In case of any dispute, all
legal matters to be settled under Delhi jurisdiction only.

First Edition: 2003

Publishing Director: RK Yadav

ISBN 81-8061-128-0

Typeset at JPBMP typesetting unit


Printed at Gopsons Papers Ltd., Sector 60, Noida
Preface

Diseases of the ear, nose and throat constitute to nearly


half of the cases attending outpatient departments in
primary care hospitals. Hence more emphasis is laid
upon teaching of the diagnosis and management of these
ailments at undergraduate level.
This handbook Short Notes and Short Cases in ENT
presents the commonly encountered diseases of ear, nose
and throat in the question answer format in a crisp and
compact manner for quick revision and recollection .The
clinical cases commonly presented at undergraduate
examination are discussed under short cases.
The book will be of immense help for final year
MBBS students preparing for their clinical examinations
and viva voce.

UN Panda
Contents
SHORT NOTES
1. The Ear: Anatomy and Physiology ............... 3
2. Diseases of External Ear and
Tympanic Membrane ...................................... 6
3. Otitis Media .......................................................... 8
4. Diseases of Middle Ear ................................. 10
5. Sound Mechanics ............................................... 14
6. Hearing Evaluation ............................................ 16
7. Tinnitus and Dizziness ..................................... 22
8. Neuro-otology ..................................................... 29
9. Paranasal Sinuses ............................................... 32
10. Rhinitis ................................................................. 36
11. Pharynx and Larynx ........................................... 39
12. Epistaxis ............................................................... 48
13. Nasal Trauma ...................................................... 50
14. Tonsils .................................................................. 54
15. Oral Medicine ..................................................... 59
16. Plastic Surgery in ENT ...................................... 64
17. Sleep Apnoea and Snoring ............................. 68
18. Disorders of Smell and Taste ......................... 71
19. Otolaryngeal Manifestations of
HIV Diseases ...................................................... 73
20. Geriatric Otolaryngology ................................. 74

SHORT CASES
Cases 1 to 10 ........................................................ 77
Short Notes
1 The Ear: Anatomy and
Physiology

The external ear consists of the auricle and external


auditory canal. Except for the lobule, the entire auricle
is composed of elastic cartilage and skin. The skin is
thin and densely adherent in front and more loosely
attached behind. The external auditory canal is
cartilaginous in outer one-third and bony in inner two-
third. The skin lining of the cartilaginous portion
contains hair follicles, sebaceous glands and cerumen
glands. The cartilage of canal is continuous with that
of auricle. Small dehiscences in the anterior portion of
this cartilage (fissures of Santorini) provide potential
path ways for spread of infection and tumor from the
canal to parotid gland. The skin lining of bony canal
is extremely thin and is continuous with the external
epithelial layer of tympanic membrane.
The middle ear or tympanic cavity is bony except for
lateral wall formed by tympanic membrane. In its
anterior aspect the Eustachian tube leads downwards
and medially to nasopharynx . In the medial aspect lies
the bony promontory that encloses basal turn of cochlea.
The middle ear opens posterosuperiorly through
auditus to mastoid antrum. The oval window is located
in a niche posterior and superior to promontory. The
round window is anterior and inferior to oval window.
The stapes fits snugly in the oval window. The handle
of malleus (manubrium) is embedded in the upper half
of tympanic membrane.
The tympanic membrane has external squamous
epithelium, the middle fibrous layer and inner mucosal
layer. It has a rounded fibrous annulus. The blood supply
of middle ear and mastoid is from branches of internal
maxillary artery. The nerve supply is through tympanic
plexus on the promontory which contains branches of
cranial nerves V,VII, IX & X. Disturbances in anatomic
regions supplied by these nerves like teeth, tongue, tonsil
and larynx may cause refered pain in the ear.
4 Short Notes and Short Cases in ENT

The Eustachian tube is cartilaginous nearest the


nasopharynx and bony nearest the middle ear. It is lined
with respiratory epithelium. The tube which is normally
passively closed is opened primarily by action of tensor
veli palatini , innervated by fifth cranial nerve. The tube
opens during swallowing but can be forcibly opened by
Valsalva. Failure of eustachian tube to open causes
negative pressure and accumulation of serous fluid in
middle ear.
The inner ear contains end organs of hearing, (the
cochlea) and equilibrium (labyrinth) both contained
within the otic capsule of temporal bone. The snail-
shaped cochlea makes 2½ turns. The bony labyrinth
contains the vestibule, the vestibular aqueduct and the
three semicircular canals.
Contained within these bony structures are the
membranes end organs which are surrounded by peri-
lymph and are filled with endolymph. Perilymph
resembles extracellular fluid while endolymph resem-
bles intracellular fluid in electrolyte composition.
Perilymph and endolymph do not communicate except
under pathological situations.
The membranous cochlea contains three compart-
ments—scala vastibulae (associated with oval window),
scala tympani (associated with round window) that
contain perilymph. The third compartment, the cochlear
duct contains endolymph and is bordered by basilar
membrane on which lie the organ of Corti containing
the hair cells that project from neuroepithelium. The
haircells are spatially organized along the basilar mem-
brane in a specific frequency responsive distribution.
Both cochlea and labyrinth are supplied by terminal
branches of basilar artery. The facial nerve passes
through internal auditory meatus between the labyrinth
and cochlea, turns and descends along medial wall of
middle ear just superior and posterior to oval window
and exits through stylomastoid foramen. It gives a
branch to stapedius which is attached to neck of stapes,
and another branch the chorda tympani that courses
through middle ear space suspended between incus and
malleus to exit near TM joint. Jacobson’s nerve, a branch
of glossopharyngeal runs in a groove across the
promontory.
The Ear: Anatomy and Physiology 5

Roughly 30 dB of energy in lost when sound passes


from air to endolymph. The transformer action of middle
ear (hydraulic action based on ratio of TM to foot plate
of stapes) recovers 27 dB. In intact ear sound activates
oval window before round window. This vibrates the
perilymph within scale vestibuli before sound reaches
the round window which overlies scala tympani. The
presence of window on both sides of basilar membrane
is important in that for a pressure wave to pass through
a relatively noncompressible liquid housed in cochlea,
there must be compliant openings at both ends of the
channel. The round window serves as a relief valve in
the bony cochlea.
6 Short Notes and Short Cases in ENT

2 Diseases of External Ear


and Tympanic Membrane

1. Six hillocks of His and their clinical significance


In the embryo, the first and second branchial arches each
give rise to three hillocks. The first arch gives rise to first
three hillocks which form tragus, helical crus and helix.
The second arch gives rise to second three hillocks
which form antihelix, scapha and lobule. When these
primitive ear hillocks fail to fuse, preauricular pits
result, that often get recurrently infected.
2. Cauliflower ear
This deformity results from cartilage loss. Subperi-
chondrial haematoma commonly due to blunt injury as
in boxers and wrestlers is responsible that deprives
cartilages of its nutrients leading to necrosis and fibrosis.
Hence subperichondrial haematoma be evacuated by
incision and properly stitched and packed.
3. Keratitis obturans
It is also known as cholesteatoma of external auditory
canal. It often causes bone erosion, it is frequently
associated with sinusitis, bronchiectasis and COPD.
Removal of the debris is the treatment.
4. Otitis externa
Pseudomonas and staphylococci are principal villains.
Symptoms include otalgia, pruritus, and foul-smelling
otorrhoea. The ear canal is erythematous with edema
often with complete obstruction. Exquisite tenderness is
common. Treatment is debridement and antibiotic-
steroid ear drops.
Malignant otitis externa is fulminant Pseudomonas
infection of ear that has medial and posterior extension
to mastoid, facial nerve and base of brain. The typical
patient is elderly, diabetic and pain is severe. Treatment
is with antipseudomonal antibiotics, local ear canal
debridement and control of diabetic state.
Diseases of External Ear and Tympanic Membrane 7

5. Otomycosis
It is fungal (Aspergillus) or candidial infection of external
ear. Pruritus is more common than otalgia. The fungal
mycelia can be seen. The ear canal shows gray white
or blackish debris that may resemble dirty cotton.
Treatment is with local instillation of clotrimazole.
6. Prussak’s space
It is superior recess of tympanic membrane. It is bound
laterally by pars flaccida, and medially by neck of
malleus. It is enclosed by the lateral malleolar fold in
the middle ear.
7. Tympanic membrane features and associated
conditions
Bulging, immobile Fluid/pus in middle ear
Retracted, immobile Obstruction to eustachian tube
Excessive mobility Healed perforation
in small area
Amber colour Serous fluid in middle ear
Blue/deep red Blood in middle ear.
8. Tympanic membrane perforation
TM has outer squamous layer, middle fibrous layer and
medial or inner mucosal layer. The outer and inner
layers regenerate to heal the perforation, not the middle
layer. This monomeric membrane (with only two layers)
is hypermobile with positive and negative pressures.
The central perforations heal quickly- daily healing rate
of 0.05 mm. Marginal perforation refuses to heal and are
difficult to repair. Growth of squamous epithelium to
middle ear cavity forms cholesteatoma, a danger of
marginal perforations.
9. Bullous myringitis
It is a viral infection, often associated with upper respi-
ratory infection. Severe otalgia is usual. Examination of
TM reveals reddish vesicles on TM that often enlarge
to form bullae. Treatment is with antibiotic steroid drops,
oral antibiotics and analgesics.
8 Short Notes and Short Cases in ENT

3 Otitis Media

1. Function of eustachian tube


It connects the middle ear cleft to nasopharynx, thus it
ventilates the middle ear and also protects middle ear
from nasopharyngeal secretions. Fluid collected in
middle ear is drained by eustachian tube.
2. Common organisms of otitis media
a. Acute otitis media—Streptococcus pneumonae, Haemo-
phillus influenzae, Moraxella catarrhalis, Gram negative
enteric bacilli (in 20% of infants) and viruses (in 4%
cases). These organisms also cause acute mastoiditis.
b. Chronic otitis media
Predominant organisms are gram-negative bacilli i.e.
E. coli, Proteus, Pseudomonas, and anaeroses like B.
fragilis. These organisms also cause chronic mastoi-
ditis.
3. Diagnosis of otitis media
• Otalgia, fever followed by otorrhoea, conductive
deafness.
• Pneumatic otoscopy is the gold standard for
evaluating position, mobility and colour of tympanic
membrane. Decreased mobility indicates effusion
or perforation and erythema indicates infection.
Retraction pockets can be seen.
4. Tympanometry in diagnosis of otitis media
Tympanometry assesses the mobility or compliance of
TM. The 5 patterns seen are (1) Type A indicates normal
middle ear pressure. (2) Type B indicates decrease
tympanic membrane mobility as in middle ear effusion.
(3) Type C demonstrates a curve with a peak <100 mm.
H2O indicating retracted ear drum. (4) Type As is a
shallow type A tympanogram seen in ossicular fixation.
(5) Type Ad is a deep A tympanogram seen in ossicular
chain disruption.
Otitis Media 9

5. Predisposition to otitis media


The eustachian tube in children is short straight (hori-
zontal) and of small caliber predisposing for easy access
of organism from nasopharynx. Minimal inflammation
can plug its lumen, aggravating the infection. Bottle-
feeding of infants particularly in supine position causes
milk reflux to middle ear through eustachian tube.
6. Indications for myringotomy and tympanostomy
tube insertion
• History of severe otitis media.
• Chronic otitis media with effusion (present for >3
months with hearing loss).
• Poor response to antibiotics.
• Impending complications (facial palsy, mastoiditis,
epidural abscess, meningitis).
• Recurrent acute otitis media (3 episodes in six months).
• Chronic retraction pockets in TM.
• Barotitis media.
• Autophony secondary to eustachian dysfunction.
Tympanostomy tube insertion into posterior superior
quadrant be avoided, as this is most compliant part of
pars tensa and may result in chronic perforation,
retraction and scarring. Insertion of tube under tympanic
annulus may cause cholesteatoma.
7. Complication of otitis media
a. Mastoiditis, petrositis, labyrinthitis, perilymphatic
fistula, facial palsy, cholesterol granuloma, choles-
teatoma, hearing loss, retraction pockets, ossicular
chain fixation/disruption.
b. Meningitis, epidural abscess, subdural empyema,
brain abscess, lateral sinus thrombosis, otitic
hydrocephalus.
8. Treatment of otitis media
Acute
Antibiotic (amoxycillin), antihistaminics, decongestant
corticosteroid (only in adult).
Chronic
Antibiotics but not antihistaminics and decongestants
• Tympanostomy tube insertion
• Adenoidectomy in children above 4 years.
Unilateral otitis media in adult may be due to naso-
pharyngeal carcinoma and demands examination of
nasopharynx.
10 Short Notes and Short Cases in ENT

4 Diseases of Middle Ear

1. Eustachian tube dysfunction


The hallmark of eustachian tube dysfunction is middle
ear effusion. Patient may experience intermittent ear
popping in absence of middle ear effusion. Those with
middle ear effusion may report otalgia, fullness in ear’s,
hearing loss and vertigo. Some patients are often asymp-
tomatic. Signs of middle ear effusion include limited
mobility of TM in pneumatic otoscopy and loss of
normal landmarks.
2. Cholesteatoma
A cholesteatoma is an epithelial cyst containing keratin.
It can be congenital or acquired. Congenital cholestea-
toma are seen in children. Potential sites include middle
ear, petrous apex and CP angle. Acquired cholesteatoma
occurs as a consequence of otitis media and Eustachian
tube dysfunction. Cholesteatoma forming on retraction
pockets due to inadequate clearance of keratin is called
primary acquired cholesteatoma while that occurring in
marginal perforation is called secondary acquired
cholesteatoma.
Presentation is with repeated infections or progres-
sive conductive hearing loss. Infection with Pseudomonas
causes malodorous discharge.
3. Complication of cholesteatoma
• Erosion of middle ear/mastoid cavity
• Labyrinthitis, facial palsy
• Semicircular canal erosion/fistula
• Meningitis, epidural/subdural abscess
• Sigmoid sinus thrombosis/phlebitis.
4. Cholesterol granuloma of middle ear
The hallmark of cholesterol granuloma is idiopathic
haemotympanum or dark bluish TM. The cholesterol
crystals induce foreign body reaction but do not cause
bone erosion.
Diseases of Middle Ear 11

5. Glomus tympanicum
It is a tiny tumor presenting as pulsatile tinnitus, seen
as a reddish blue mass behind TM, often confused with
extension of glomus jugular to middle ear. CT scan can
differentiate the two and excision is curative.
6. Otosclerosis—Clinical features
Fixation of foot plate of stapes to oval window causes
conductive hearing loss. Patients are between 30 to 50
years of age and usually seek advice 2 to 3 years after
symptomatic hearing loss. Vertigo occurs in half.
Disease is usually unilateral and recruitment is absent
with normal speech discrimination. The bone conduc-
tion curve shows Carhart notch. Positive family history
is present in half . Sensory neural deafness occurs in
advanced disease.
7. Cochlear reserve
Otosclerosis falsely decreases sensory neural levels on
audiometry. Hence surgery on stapes in otosclerosis not
only improves conductive loss but also sensory neural
levels. Cochlear reserve refers to true sensory neural
thresholds in a patient with otosclerosis prior to surgery.
The sensory neural levels prior to surgery may be
estimated by adding 5dB at 500 Hz, 10 dB at 1000 Hz,
15 dB at 2000 Hz and 20 dB at 4000 Hz.
8. Ossicular discontinuity
It occurs when the incudostapedial joint is separated
completely. Maximum conductive hearing loss occurs
when the TM is intact. Patient has better hearing if
perforation is present.
9. Sodium fluoride in otosclerosis
Sodium fluoride improves sensory neural hearing loss
or vestibular symptoms, but not the conductive
deafness.
10. Causes of middle ear effusion
• Acute otitis media- exudation.
• Failure of middle ear clearance- ciliary dysfunction,
mucosal oedema, increased viscosity of secretions,
change in, nasopharyngeal or middle ear pressure
gradient.
• Barotraumas—decreased middle ear pressure cau-
sing transudation.
12 Short Notes and Short Cases in ENT

• Eustachian tube dysfunction due to adenoids,


nasopharyngeal scarring, cleft palate and submucous
cleft.
11. Differential diagnosis of otogenic ear pain
External ear Middle ear/mastoid
Furunculosis ACOM
Malignant otitis externa Acute mastoiditis
Cerumen impaction Barotrauma
Perichondritis Acute Eustachian tube dysfunction
Bullous myringitis Petrositis
Herpes zoster Subperiosteal abscess
Otomycosis Trauma
Keratosis obturans Extradural abscess
Cholesteatoma of ear canal Lateral sinus thrombosis

12. Otorrhoea
1. From external ear canal
• Cerumen accumulation with infection
• Otitis externa (Pseudomonas)
• Impacted foreign body
• Canal trauma
• Otomycosis
• Dermatosis (allergic seborrhea, psoriasis)
• First branchial cleft cyst.
2 From middle ear
• ACOM with perforation
• CSOM with perforation/cholesteatoma
• Tuberculosis of middle ear
• Neoplasm.
13. Aural pruritus
• Otitis, otomycosis
• Otorrhoea of any cause
• Dermatosis (Psoriasis, seborrheic dermatitis, contact
dermatitis)
• Medical conditions with generalized pruritus—renal
failure, diabetic, lymphoma, chronic hepatic disease.
• Psychoneurosis
Infectious Diseases 13

14. Causes of tinnitus


Subjective Objective
Presbycusis AV malformation
Meniére’s disease Glomus tumor
Noise induced hearing loss Vascular loops
Otosclerosis Persistent stapedial artery
Dysthyroid state Dehiscent jugular bulb
Multiple sclerosis Palatomyoclonus
Postmeningitis Patulous eustachian tube
Drugs—aspirin, NSAID Stapedial muscle spasm
aminoglycosides, diuretics Arnold Chiari malformation
alcohol Hypertension
TM joint disease Carotid artery stenosis
Psychogenic
14 Short Notes and Short Cases in ENT

5 Sound Mechanics

Sound Mechanics in External and Middle Ear


The external ear has a resonance frequency of 3 to 4 Hz.
The tympanic membrane and ossicular chain transmit
most efficiently frequencies between 500 to 3000 Hz
with resonance around 1 KHz. The effective vibrating
area of TM is about 17 times the area of footplate of stapes
resulting in 17 times increase in sound energy. The
handle of malleus is about 1.3 times the length of short
process incus, so the force at the stapes is increased by
1.3 fold; all this contribute to a 25 dB increase in sound
energy arriving at cochlea.

Anatomy and Tonotopic Organization in Cochlea


The snail-like 2 ½ turn cochlea has three compartments:
the top—scala vestibuli, the middle—scala media and
the bottom—scala tympani. Reissner’s membrane sepa-
rates the scala vestibuli from scala media and basilar
membrane separates scala media from scala tympani.
Endolymph, the fluid similar to intracellular fluid fills
scala media, whereas perilymph–similar to extracellular
fluid fills scala vestibuli and scala tympani. The scala
media houses organ of Corti, that contains hair cells
sitting on basement membrane and are overlaid by
tectorial membrane. Cochlea has one layer of inner hair
cells and 3 to 4 layers of outer hair cells spiraling up
the central axis. Higher frequency sounds are detected
as base of cochlea while lower frequency sounds are
detected at the apex. The inner hair cells have afferent
innervation and the outer hair cells have efferent
innervation. The tectorial and basilar membranes are
connected centrally. Sound moves these two structures
differentially, producing a shear force that bends the
stercocilia. Movement of stercocilia opens and closes ion
channels producing receptor potential in the inner cells
which in turn releases neurotransmitter onto afferent
nerve fibers, signaling the brain to the presence of a
Sound Mechanics 15

specific sound frequency. The specific hair cells which


are stimulated by a given sound depend on the
tonotopic map of the basilar membrane. The afferent
impulse passes through cochlear nuclei, superior olive,
lateral lemniscus, inferior colliculus and the medial
geniculate body to reach auditory cortex.

Vestibular Endorgans and their Stimulation


The vestibular apparatus consists of the utricle, saccule
and the three semicircular canals placed at right angles
to each other. The semicircular canals detect angular
acceleration like head rotation. The utricle has receptors
cells in horizontal plane while saccule has receptor cells
in vertical plane. They detect linear acceleration.
Semicircular canals contain hair cells with sterco-
cilia embedded in jelly like cupola. Angular motion
moves the cupola bending the stercocilia, thus stimu-
lating the hair cells. An otolithic membrane made of
calcium carbonate overlies the hair cells of utricle and
saccule. This membrane is denser than endolymph.
Therefore gravity and linear acceleration move the
membrane relative to hair cells, bending the cell’s
stercocilia and thereby generating an impulse.
16 Short Notes and Short Cases in ENT

6 Hearing Evaluation

1. Weber’s test
256/512 Hz tuning fork is used for the purpose. The
vibrating tuning fork is placed on center of forehead. In
conductive hearing loss patient has better appreciation
of vibration in the affected ear. In sensory hearing loss,
patient appreciates vibration better in healthy ear.
Patients with equal hearing or bilaterally symmetrical
hearing problem will localize the sound to the center of
forehead.
2. Rinne test
It is also used to differentiate between conductive and
sensory neural hearing loss. The test is performed by
alternatively placing the prongs of a vibrating 256 Hz
tuning fork at the patient’s ear canal and the base of
the tuning fork on patient’s mastoid bone. In a patient
with normal hearing and middle ear the tuning fork is
heard louder at the ear canal or equally loud in both
positions. Similar is the finding in sensory neural
hearing loss. Patients with conductive loss, however,
hear the tuning fork sound louder at the mastoid
position (negative Rinne test). A negative test is obtained
with at least hearing loss of 25 dB.
3. Schwabach’s test
It is a crude estimation of sensory neural deficit. The base
of vibrating tuning fork is placed on mastoid. Once
patient is unable to perceive the decaying vibrations,
examiner puts the tuning fork on his own mastoid. If
examiner still hears the vibration, the patient is having
sensory neural hearing loss.
4. Audiogram
An audiogram is a graphic representation of auditory
threshold responses which are obtained from testing a
patient’s hearing with pure tone stimuli. The typical
audiogram is determined by establishing hearing
Hearing Evaluation 17

thresholds for single frequency sounds at 250, 500, 1000,


4000 and 8000 Hz. Human ear can detect frequencies
between 20 to 20,000 Hz. The speech frequency spectrum
ranges 400 to 3000 Hz. The hearing threshold level
(HTL) of normal hearing is –10 to 20 dB. HTL of 20-40
dB is mild hearing loss, 40 to 60 dB is moderate hearing
loss, 60 to 80 dB is severe hearing loss and > 80 dB is
profound hearing loss.
5. Air bone gap
An air bone gap is the difference in decibels between
the hearing threshold levels for air and bone conduc-
tion. Significant air bone gaps represent conductive
hearing loss. With sensory neural loss the air and bone
conduction thresholds are approximately equal.
6. Role of tensor tympani and stapedius
They dampen middle ear mechanics. Loud sounds
(>80dB) cause stapedius contraction, thus limiting
movement of stapes, thus serving a protective role.
Stapedius contraction also occurs during chewing and
vocalization, and thus may reduce self-generated noise.
Stapedius always contracts bilaterally, even when only
one ear is stimulated.
7. Acoustic reflex neural pathway
The acoustic reflex has both ipsilateral and contralateral
pathways but the majority of neurons run through
former. The ipsilateral pathway begins at cochlea and
proceeds through 8th nerve, cochlear nucleus, trapezoid
body, superior olivary complex and facial motor nucleus
to ipsilateral stapedius muscle. The contralateral
pathway crosses brainstem to opposite cochlear nucleus
and follows the path like opposite side to end in
contralateral stapedius muscle. The acoustic reflex is
measured by immitance meter.
8. Tests for differentiating cochlear from retroco-
chlear deafness.
It is done by speech audiometry because speech
recognition requires greater synchronous neural firing
that is necessary for appreciation of pure tones. Speech
reception threshold (SRT) is the intensity at which
speech is recognized as a meaningful symbol. It is
obtained by presenting two syllable words with an
equal accent and the intensity at which patient can
18 Short Notes and Short Cases in ENT

repeat 50% of words in his SRT. Patient’s word discrimi-


nation ability is tested by presenting one syllable words
at 25 to 40 dB above SRT. Deterioration in discrimination
ability at higher intensities above SRT indicates lesion
in 8th nerve or central auditory pathways.
9. Decibel (dB)
A dB is equal to 20 times the logarithm of the ratio of
the sound pressure required to achieve threshold in the
patient to the sound pressure required to achieve
threshold in normal person. Therefore a change in 6 dB
represents doubling of sound pressure and a change of
20 dB represents a 10-fold change in sound pressure.
Loudness which depends on the frequency, intensity
and duration of a sound, doubles with approximately
each 10 dB increase in sound pressure level.
10. Auditory brainstem response (ABR)
ABR is an objective physiologic measurement of
hearing. The electrical activity is picked up by scalp
electrodes in response to clicks delivered to patient
through ear phones. The electrical activity has 5 latency
specific wave peaks, each peak corresponding to a site
in the auditory pathway. Hence ABR can detect lesions
in hearing pathway and is useful for testing hearing in
infants and young children. (1) Wave I- 8th nerve action
potential (2) Wave II- cochlear nucleus (3) Wave III-
olivary complex (4) Wave IV- lateral lemniscus (5) Wave
V-inferior colliculus.
11. Hearing evaluation in pediatric patients
• Behavioral observation audiometry—reaction of
child to sounds of various intensities.
• Play audiometry—a game is incorporated into the
test.
• Speech audiometry for estimation of SRT.
• Immitance measurements.
• ABR.
12. High risk factors for hearing loss
• Family history of childhood hearing impairment.
• Congenital perinatal infections (CMV, HSV, rubella,
toxoplasma, syphilis).
• Birth wt < 1500 gm.
• Kernicterus.
• H. influenzae meningitis.
Hearing Evaluation 19

• Severe birth asphyxia—Apgar score 0 to 3.


High risk newborns should be screened for hearing
with ABR prior to hospital discharge.
13. Causes of conductive hearing loss
External ear—Cerumen impaction, foreign body, exos-
tosis, osteoma
Middle ear—TM perforation, tympanosclerosis, retracted
TM, otitis media, haemotympanum, eustachian tube
dysfunction, otosclerosis and ossicular chain disrup-
tion.
14. Major causes of sensory neural hearing loss
• Hereditary (Alpert syndrome)
• Noise induced
• Presbycusis
• Multiple sclerosis
• Ototoxicity
• Diabetes mellitus
• Meniere’s disease
• CP angle tumors
• Acoustic trauma
• Labyrinthitics
• Barotrauma
All these lesions can be grouped as hereditary, dege-
nerative, toxic, infection, immune mediated, traumatic,
etc.
15. Presbycusis
It is a slowly progressive symmetric sensory neural
hearing loss. Hearing loss is the greatest in frequencies
>20,000 Hz with a significant decrease in speech dis-
crimination. Common to people beyond 60, often they
can hear conversation but cannot interpret the words.
16. Drug Ototoxicity
Aminoglycosides damage cochlear hair cells; loop
diuretics (ethacrynic acid) damage stria vascularis; the
specialized epithelium in organ of Corti responsible for
maintaining ionic balance. Patients receiving more than
one ototoxic drugs or those with low GFR are at greatest
risk. Aminoglycosides first affect high frequencies but
later the low frequencies as the hair cells at basal
cochlea are affected first and subsequently those at
cochlear apex.
20 Short Notes and Short Cases in ENT

Sudden sensory neural hearing loss (SSNHL)


Only 10 to 15% of these patients have a specific etiology.
Most are attributed to infections, vascular or otologic
membrane rupture. Cochlea receives entire blood supply
from cerebellar artery; thrombotic/embolic events can
cause SSNHL. Barotrauma from diving or ascent can
cause fistula in round/oval windows, rupture of basilar/
Reissner’s membrane with mixture of endolymph and
perilymph.
Those cases of SSNHL with minimal hearing loss,
low frequency loss without vestibular symptoms have
better chances of recovery. Idiopathic SSNHL may
respond to corticosteroid.
17. Types of analog hearing aids
(1) Behind the ear (2) In the ear (3) In the canal (4)
Completely in the canal (5) Body aids (6) Eyeglass aids
(7) CROS aids.
18. Advantages of behind-the-ear aids
1. Best for patients with moderate to severe hearing loss.
2. Large enough to accommodate multiple controls.
3. Microphone and receiver are more easily separated
which allow for less feedback.
19. CROS type hearing aid
CROS stands for contralateral routing of signals. This
is helpful in those with useful hearing in one ear, but
no or unaidable hearing in other ear. A microphone is
placed on the side of patient’s poorer ear and the signal
received is routed to opposite ear and amplified. These
aids improve the patient’s ability to hear sounds that
originate on the side of the poor hearing ear.
20. Bilateral hearing loss—hearing aids for
Patients of bilateral hearing loss do better with binaural
amplification that eliminates the shadow effect, i.e. 6 dB
loss in sound intensity when sound has to cross the
head to contralateral ear. It also provides better speech
discrimination, improved ease of hearing, heightened
speech localization.
21. Cochlear implant
Cochlear implant is indicated for patients over 1½ to 2
years of age who have profound binaural sensory neural
hearing loss with intact 8th nerve function. The behind
Hearing Evaluation 21

the-ear-microphone receives sounds and converts it into


electrical signals which are delivered to external signal
processor worn on belt. The signal processor modifies
the signals and delivers it to transmitter over the mastoid
which in turn delivers the signal to the implanted
receiver/stimulator directly or indirectly. Directly, the
signal may be carried via a hard-wired percutaneous
connector. Indirectly, the signal may be carried by an
FM radiofrequency or magnetic induction. The receiver/
stimulator implanted under the skin in the mastoid
further modifies the signal and delivers it to electrodes
implanted in the scala tympani. These electrodes
stimulate the spiral ganglion cells in the cochlea.
22. Assistive listening/living devices
Assistive listening devices are systems that improve
listening in noise and other difficult listening situations.
These include frequency modulation systems to be worn
by young children with profound loss. Infrared listening
system are used in theaters to assist deaf person
attending public events. Assistive living devices make
living easier for the deaf. They include telephone
amplifiers, teletypewriters, and devices that alert deaf
person to signals like door bell, smoke alarm, baby’s cry,
etc.
22 Short Notes and Short Cases in ENT

7 Tinnitus and Dizziness

Tinnitus or buzzing/ringing in ear is a sound sensation


that originates in head. It can be objective when it can
be heard by an observer or subjective that defies detec–
tion. Tinnitus can also be classified as vasculogenic,
myogenic, peripheral, sensory neural, central, external
and middle ear toxic, etc.
Glomus tumor behind TM, AV malformations
around ear, venous hum, can cause vascular tinnitus.
Venous hum is seen in high cardiac output states and
in jugular vein compression by second cervical vertebra.
Compression of jugular vein abolishes the tinnitus.
Cerumen, FB in external ear can cause tinnitus. Palatal
myoclonus (often associated with multiple sclerosis,
ICSOL, CVA) can cause tinnitus. Salicylates and NSAID
cause hearing loss and tinnitus which is bilateral. The
sensory neural hearing loss is mild (20-40 dB) but reverts
to normal on discontinuation of the drugs. Aminogly-
cosides cause tinnitus, deafness and vertigo. Tinnitus
may precede hearing loss in most patients but this is
not absolute. Meniere’s disease, otosclerosis, middle ear
effusion can have tinnitus so also CP angle tumors (in
10%).
1. Drugs causing tinnitus
Besides alicylates, NSAIDs, aminoglycosides, anti–
malarials, a host of drugs like ACE inhibitors, antibiotics,
antidepressants, antihistamines, betablocker, calcium
channel blockers, tegretol, alopurinol, lilthium, hepatitis
B vaccine, sulfasalazine, buspiron are known to cause
tinnitus.
2. Masking of tinnitus
Eighty percent of patients will have their tinnitus
masked by 6 dB or less of sound. White noise or tinnitus
in night in absence of background noise can be relieved
by nonidentifiable background sound, hum of a fan,
Tinnitus and Dizziness 23

non-operational frequency of radio set, etc.


3. Treatment of tinnitus
• Correction of etiologic factors
• IV xylocaine for short-term relief
• Biofeedback.
4. Dizzines and Vertigo
CNS haemorrhage and infarcts are the most serious
emergent causes of dizziness.
Bacterial labyrinthitis, acute perilymphatic fistula,
trauma to temporal bone are also important. Vertebro–
basilar TIAS, acoustic neuroma, Meniére’s disease
multiple sclerosis, CNS vasculitides are responsible in
few. Dizziness be distinguished from unsteadiness or
light headedness which is due to anemia, orthostatic
hypotension, etc. Vertigo is the illusion that the patient’s
body or environment is spinning or tumbling. However,
benign paroxysmal positional vertigo remains the most
common cause of dizziness. Hypoglycaemia and other
metabolic disturbances cause unsteadiness mimicking.
5. Duration of vertigo
Seconds BPPV
Minutes Migraine, vertebrobasilar TIA
Hours- Meniere’s disease, syphilis
Days- Vestibular neronitis, infarct of labyrinth
(vasculitis).
6. Mandatory neurological exam in dizzy patient
• Evaluation of cranial nerves
• Cerebellar function
• Nystagmus
• Hearing.
7. General categories of nystagmus
Gaze-evoked nystagmus—Nystagmus appears when
patient looks left/right, up/down.
Positional nystagmus—occurs with certain head or body
positions.
Spontaneous nystagmus—occurs without stimulation.
Induced nystagmus—elicited by caloric test/rotational
movements.
8. Investigations in a dizzy patient
• Audiogram and electronystagmogram
24 Short Notes and Short Cases in ENT

• ABER
• MRI with gladolinium contrast
• CT is preferred if temporal bone lesion suspected
• CBC, VDRL and lipid profile.
9. Difference between central vs peripheral vartigo
Peripheral Central
Usually paroxysmal Seldom paroxysmal
Severe Mild
Short duration Prolonged duration
Deafness common Deafness rare
Tinnitus common Tinnitus rare
Vomiting marked Vomiting rare
Head position Head position has
has influence no influence
Other neurologic Other neurologic
dysfunction absent dysfunction present

10. Electronystagmogram (ENG)


This is an electronic method of evaluating the visual
tracking and vestibular systems. Electrodes on patient’s
face record eye movements by recording the changing
electrical potential between the cornea and retina.
Abnormal eye movements may be demonstrated when
the patient is subjected to a series of visual tracking
tests, caloric irrigation and positional maneuvers.
11. Caloric responses
Normally when the left ear is stimulated with cold water
the eyes turn to left but fast phase of nystagmus occurs
to right. Stimulation with warm water yields opposite
results, i.e. nystagmus is cold = opposite, warm= same
side, abbreviated as COWS.
12. Posturography
It is a test of vestibulospinal system in which standing
sway is measured with a force plate. However, it does
not localize disease to peripheral or central vestibular
system. It is most useful in vestibular rehabilitation
programmes.
13. Meniere’s triad
It is (1) fluctuating lowtone sensory neural hearing loss,
(2) fluctuating tinnitus and (3) episodic vertigo. Feeling
of aural fullness is also common.
Tinnitus and Dizziness 25

14. Benign paroxysmal positional vertigo (BPPV)


It is sudden episodic vertigo precipitated by head move-
ments like rolling over in bed, looking upward; each
episode lasting < 1 minute. Hearing loss and tinnitus
are absent. It is self-limited and resolves spontaneously
over weeks to months.
BPPV is caused by presence of heavy particles in one
of the semicircular canals, possibly otoliths that have
been displaced from utricle. Movement of these particles
with gravity causes stimulation of the canal. Since the
canal only senses spinning motion, gravity is misinter-
preted as spinning. Therapeutic head maneuvers that
rotate the particles out of the affected canal may help.
The Eply maneuver has a success rate of 90%. Vestibular
rehabilitation programmes are also helpful. When none
is helpful, surgical ablation of the ear or blockage of
affected canal will stop the spell.
15. Pathophysiology of Meniere’s disease
It is due to fluctuating volume/pressure changes within
the closed fluid system. Normally, endolymph moves
from the cochlea, where it is produced, to the
endolymphatic sac where it is absorbed. Any disruption
in this process can lead to hydrops that disturbs
cochlear duct and produces vertigo and hearing loss.
The vertigo attacks may occur anywhere, anytime
and may awaken the patient from sleep. As disease
progresses, the tinnitus becomes constant and often
irritating chief complaint. Severe hearing loss super-
venes and then vertigo disappears. About one-third of
patients develop bilateral disease.
16. Surgery in Meniere’s disease
Decompression of hydrops sac may achieve variable
success. Vertigo control is the primary aim of therapy.
When hearing loss is prominent-labyrinthectomy is very
helpful. When patient has useful hearing-vestibular
nerve section and transtympanic aminoglycoside
injection may benefit.
17. Perilymphatic fistula
Most commonly these fistulas occur in the oval or round
windows of the middle ear, allowing the perilymph to
enter middle ear. Classically the patient gives history of
barotrauma, head trauma or penetrating injury to
26 Short Notes and Short Cases in ENT

tympanic membrane. Patient has vertigo, tinnitus and


hearing loss. Often subsiding at rest, these symptoms
may be precipitated with straining like sneezing/nose
blowing. Haemotympanum may be evident. A fistula
test in which pressure is raised in external auditory
canal may cause dizziness. However, diagnosis is
strongly based on history. A definitive diagnosis cannot
be made until surgical exploration.
Most fistulas close spontaneously. The patient
should keep the affected ear above the level of heart and
elevate the head end of bed by 4-6”. Straining, coughing
and lifting be avoided. Surgical treatment is of cleaning
of middle ear dry and packing it with gelfoam, fascia
or fibrous tissue.
18. Aminoglycoside ototoxicity
Aminoglycoside ototoxicity occurs in 5 to 10% cases,
causing sensory neural hearing loss, tinnitus and ver-
tigo. Amikacin and neomycin cause more of cochlear
toxicity while gentamicin, tobramycin and streptomycin
cause more of vestibular toxicity, particularly causing
disequilibrium and oscillopsia where objects may
appear to jump or bob spontaneously or with head
movement. Head movement dependent oscillopia is
classic symptom of aminoglycoside toxicity.
19. Vertebrobasillar insufficiency
Vertebrobasillar insufficiency causes transient vertigo
accompanied by brainstem symptoms like diplopia,
dysarthria, dysphagia, visual hallucination, etc. Drop
attacks are common. Symptom can be provoked by
hyperextension of the neck. Vertigo of cerebrovascular
insufficiency can be due to cerebellar ischemia, ischemia
of lateral medulla where vestibular nuclei are placed or
ischemia involving labyrinthine artery. Tumors at CP
angle can cause vestibular symptoms so also gliomas
and secondary tumors of brainstem and cerebellum.
20. Motion sickness
Motion sickness or kinetosis is not a disease but rather
a physiologic response to a mismatch between vestibular
and visual information about the moving environment.
It is characterized by nausea, vomiting, pallor and
sweating.
Tinnitus and Dizziness 27

21. Cogan’s syndrome


It is a systemic autoimmune disorder affecting the inner
ear and the eye. Vestibuloauditory symptoms are severe
and bilateral.
22. Vascular loops and vertigo
Vascular loops in CP angle can cause vertigo, commonly
arising from anterior inferior cerebellar artery and intru-
ding into internal auditory meatus, causing compres-
sion of 8th nerve. If patients who suffer from brief attacks
of positional vertigo and tinnitus are responsive to car-
bamazepine, vascular loop causing compression is
likely.
23. Pressure changes with diving and flying
Pressure is double at 10 meter below surface and this
decrease is linear so that 10 meter of descent causes
pressure increase of 1 atom. Air pressure at 18000 feet
is one half of that at sea level.
Normally the eustachian tube is closed, only opening
when there is positive pressure in nasopharynx or by
contraction of tensorveli palatini, levator palatini or
salpingo pharyngeus. During descent or ascent, if
eustachian tube fails to open to equalize pressure in
middle ear and nasopharynx, there will be otalgia due
to contraction/expansion of air in middle ear cavity
respectively due to increased or decreased external air
pressure. Ear pain is more common while diving down
than ascending up.
24. Innervations of the ear
External ear is innervated by auriculotemporal (branch
of trigeminal), facial and auricular branch of vagus
(Arnold’s nerve). The middle ear is supplied by
Jacobson’s nerve (IX), auriculotemporal and Arnold’s
nerve.
25. Children and flying
Children with common cold should not fly because the
eustachian tube is of narrow caliber, tensor velipalatini
is poorly functioning and eustachian tube function is
compromised because of the viral infection. During
ascent the volume of air in middle ear expands and the
TM bulges out. Once pressure differential reaches 90
mm Hg eustachian tube is locked and further pressure
rise leads to rupture of tympanic membrane.
28 Short Notes and Short Cases in ENT

26. Barotrauma
Barotrauma can cause haemotympanum, rupture of TM,
rupture of round/oval window with perilymphatic
fistula and disruption of inner ear with vertigo, tinnitus
and sensory-neural hearing loss. Decompression of
inner ear (bubling of nitrogen with vessel occlusion) can
have similar presentation and responds to hyperbaric
oxygen.
8 Neuro-otology

1. True neoplasms of temporal bone and external ear


canal
Fibrous dysplasia, Langerhan’s cell histiocytosis,
leukemia, sarcoma.
2. True neoplasms of middle ear
Glomus tumor or paragangliomas are the most common
followed by cholesteatoma, squamous cell carcinoma,
rhabdomyosarcoma. Pulsatile tinnitus is a characteristic
feature of glomus tumor. On pneumatic otoscopy,
positive pressure may cause blanching of the mass
under TM (Brown’s sign). Because of appearance it may
be mistaken for high riding jugular bulb or aberrant
carotid artery. The tumor is benign but conductive loss
is usual.
3. Jugular foramen syndrome
Jugular foramen structures (IX,X,XI cranial nerves) may
be affected due to lymphadenopathy, tumors and skull
fractures involving jugular foramen. The most common
tumors at this location are paragangliomas, Schwan-
nomas and sarcomas.
4. Lesions affecting petrous apex
Inflammatory—cholesteatoma cholesterol granuloma,
mucocele.
Infectious—osteomyelitis (Petrous apicitis).
Neoplastic—Schwannoma, meningioma, glomus tumor,
chordoma.
Aneurysm—aneurysm of intrapetrous carotid artery.
5. Cholesterol granuloma
It is a temporal bone inflammatory lesion associated
with giant cell reaction. It may be associated with otitis
media, barotraumas or cholesteatoma. Common presen-
tation is with pain, facial nerve or vestibulo-cochlear
nerve dysfunction.
30 Short Notes and Short Cases in ENT

6. Anatomic segments of facial nerve


Intracranial—from brainstem to internal auditory canal
Meatal—from fundus of internal auditory canal to
meatal foramen (narrowest aperture of facial nerve’s
bony canaliculus).
Labyrinthine—(Narrowest segment of facial nerve) from
meatal foramen to geniculate ganglion.
Tympanic—coursing adjacent to oval window to the
pyramidal eminence of stapedius tendon (second genu)
15-30% normal nerves may be dehiscent in this segment.
Mastoid—(vertical segment) from second genu to
stylomastoid foramen.
Extratemporal—from stytomastoid foramen to facial
muscles.
7. Facial nerve repair/decompression
Facial nerve decompression is warranted when nerve
weakness is progressive and is >90% as determined
from Hilger facial nerve stimulator or electroneuro-
graphy. This is particularly beneficial in Ramsay Hunt,
Bell’s palsy or perigeniculate injury due to trauma. The
site of lesion can be determined from history, CT/MRI.
Decompression of labyrinthine and meatal segments is
best approached via middle fossa. Transmastoid
approach is for decompression of tympanic segment.
End to end anastomosis is best but when not possible
graft from sural or greater auricular nerve can be used.
Hypoglossal facial anastomosis can give acceptable
result when proximal stump of facial is not identifiable.
8. Structures in internal auditory canal
Anterior superior quadrant—facial nerve.
Anterior inferior quadrant—cochlear nerve
Posterior superior quadrant—superior vestibular nerve
Posterior inferior quadrant—inferior vestibular nerve.
9. Cerebellopontine angle (CPA)
CPA is a potential space in posterior fossa, anteriorly
bounded by temporal bone, posteriorly by cerebellum,
inferiorly by cerebellar tonsils and superiorly by pons
and cerebellar peduncles. Facial and vestibulocochlear
nerves travel superiorly and laterally through CPA and
into internal auditory canal. Acoustic neuromas account
Neuro-otology 31

for 80% CPA tumors; rest being meningioma, lipoma,


epidermoids, cholesteatoma, arachnoid cysts, aneury-
sms.
10. Acoustic neuroma
This is benign encapsulated tumor of VIII nerve sheath,
arising from vestibular nerve twice as common than
auditory nerve. 70% of them grow slowly but 30%
remain stable and 2% of population have silent acoustic
neuromas in MRI. Most frequent presentation is tinnitus
with high frequency unilateral sensory neural hearing
loss. Facial nerve weakness and cerebellar symptoms
may be present. Numbness of posterior aspect of concha
is suggestive since it is innervated by facial nerve.
Vertigo is not a common complain because of adap-
tation consequent to slow growth of the tumor. Loss of
discrimination disproportionate to pure tone results and
loss of stapedial reflex in impedance audiometry demand
ABR and contrast MRI to exclude acoustic neuroma.
ABR shows wave V to be absent or prolonged.
Besides surgery- gamma knife and linear accelerator
can be employed for acoustic neuroma. Gamma knife is
suitable when tumor is < 3 cm. Chances of facial nerve
damage and hearing loss are low.
32 Short Notes and Short Cases in ENT

9 Paranasal Sinuses

Anatomy
There are 4 pairs of nasal sinuses—the frontal, maxil-
lary, ethmoid and sphenoid. The osteomeatal complex
includes the uncinate process, maxillary ostium, middle
turbinate, bulla ethmoidalis and ethmoid infundibulum.
The frontal, ethmoid and maxillary sinuses drain
through this area. Hence any mucosal thickening or
congenital variation in this complex may cause
obstruction, stasis and infection of upstream sinuses.
Endoscopic sinus surgery focuses on maintaining
normalcy of this complex/unit. The nasolacrimal duct
opens into inferior meatus; the maxillary sinus, frontal
sinus and anterior ethmoidal cells drain to middle
meatus and the sphenoid sinus and posterior ethmoidal
cells to superior meatus. Symmetry is the rule for maxil-
lary sinus but the frontal sinus vary widely in shape
and in 5% population one frontal sinus may remain
undeveloped. An ethmoid sinus in adult has 10-15 cells
with total volume of 15 ml, similar to maxillary sinus.
Volume of frontal and sphenoid sinus is around 7 ml.
Sphenopalatine artery, a branch of internal maxillary
artery enters nasal cavity through sphenopalatine
foramen to supply septum, lateral wall of nose and
ethmoid sinus. The anterior and posterior ethmoidal
arteries are branches of ophthalmic artery in orbital
cavity and leave orbit in their foramina in medial orbital
wall to supply ethmoid sinus. The PNS are innervated
by sensory branches of trigeminal nerve. Postganglionic
parasympathetic secretomotor fibers from pterygo-
palatine ganglion also supply mucosa of maxillary
sinus. Though the real function of sinuses is unknown,
they cause humidification and warming of inspired air,
lighten the weight of skull, improve vocal resonance,
absorb shock to face and skull, increase area of olfactory
membrane, regulate intranasal pressure and secrete
Paranasal Sinuses 33

mucus that keeps the nasal chambers moist. They are


lined by ciliated, pseudostratified columnar epithelium
containing goblet cells and mucoserous glands. In the
maxillary sinus, secretion transport starts from the floor
of the sinus in a stellate pattern but the frontal sinus
has a flow of secretions both into and out of the sinus.
Agger nasi cells are the most anterior of anterior
ethmoidal cells and are in close proximity to frontal
recess, often obstructing outflow from frontal sinus.
Haller cells are ethmoidal cells that have extended into
maxillary sinus around its ostium. They are found in
10% population and obstruct maxillary sinus drainage
and ventilation. Onodi cells are posterior ethmoidal
cells extending along sphenoid sinus. The optic nerve
may lie within an Onodi cell.
The term conchabullosa refers to pneumatization of
middle turbinate, usual in 30% of population. Its
enlargement obstructs PNS ventilation and mucociliary
clearance.

Sinusitis
• Acute, subacute and chronic sinusitis refer to
symptoms lasting <4 weeks, 4-12 weeks and >12
weeks respectively.
• Three major factors contributing to sinusitis are (1)
patency of sinus ostia (2) failure of normal mucous
transport (3) change in quality of sinus secretion.
Patency is hindered by obstruction mostly due to
mucosal edema or anatomic abnormality leading to
decreased O2 tension, and bacterial growth. Mucous
transport is hindered by ciliary dysfunction
consequence to infection or allergens.
• Most common cause of acute bacterial sinusitis is
recurrent viral upper respiratory infection; others
include allergy, barotraumas, foreign body, immune
deficiency (fungal sinusitis).
• Chronic bacterial sinusitis is due to allergic rhinitis,
nasal polyposis, nasal septal deviation, cystic
fibrosis, ciliary dysfunction (Kartagener’s syndrome.
Young’s syndrome).
• Seventy per cent cases of acute sinusitis is by
Streptococcus pneumonas and H influenzae, only 6-10%
is by anaerobes. M catarrhalis is implicated in 20%
cases in children. Most of these organisms are lacta-
mase producing.
34 Short Notes and Short Cases in ENT

• Anaerobes play the major role in causation of


chronic sinusitis. Aspergillus is the most common
cause of invasive and noninvasive fungal sinusitis.
• Symptoms of sinusitis are not very specific, but fever,
headache, nose block, rhinorrhoea, post-nasal drip,
persistent cough and bronchospasm are the pointers.
Tenderness over the affected sinus and congestion
of the middle turbinate and uncinate process in
infection of anterior sinuses may be seen.
• Nasal endoscopy may clinch the diagnosis from
visualization of osteomeatal complex. In sinus X-ray
air fluid level and sinus opacification are highly
suggestive. Most commonly obtained films include
occipitomental (Waters), occipitofrontal (Caldwell)
and lateral views. Coronal CT allows better visuali-
zation of osteomeatal complex and ethmoidal
sinuses. CT is indicated in evaluation of refractory
sinusitis, complication of sinusitis and suspected
malignancy. MRI yields better soft tissue resolution
and can better distinguish fungal sinusitis, sinus
neoplasia and intracranial extension. Since there is
no radiation it is preferred for children.
• Periapical and periodontal disease may lead to
sinusitis so also oroantral fistula occurring after
dental extraction.
• Complication of sinusitis may be of orbit or
intracranial. Orbital complications include edema of
eyelids, orbital cellulites (proptosis, chemosis,
restricted eye movements), subperiosteal abscess,
orbital abscess (Severe proptosis, complete ophthal-
moplegia) and cavernous sinus thrombosis.
Intracranial complications are meningitis, epidural
abscess, subdural abscess, brain abscess, intracranial
thrombophlebitis and venous thrombosis.
• Mucocele occurs in chronic sinusitis due to osteal
obstruction and involve frontal, sphenoid and ante-
rior ethmoid, sinuses. They can invade surrounding
structures or can be infected. Pott’s puffy tumor, the
doughy swelling on forehead is due to osteomyelitis
secondary to frontal sinusitis.
• Though 40% cases of acute bacterial sinusitis
spontaneously resolve, antibiotic therapy hastens
recovery, prevents complications and prevents pro-
gressive mucosal changes that may result in chronic
Paranasal Sinuses 35

sinusitis. In uncomplicated cases amoxycillin for 10-


14 days is sufficient but azithromycin, cefuroxime
and cefachlor are equally effective. When betalac-ta-
mase producers are suspected amoxycillin- clavu-
lanic acid or clindamycin may be preferred. Upto
44% of anaerobes are also betalactamase positive.
• Topical decongestants and topical corticosteroids
should be administered in Mecca position or be
inhaled for 5 days to reduce nasal inflammation and
mucosal swelling, there by improving sinus
drainage. Mucolytics (guaifesin) has a positive role.
Oral antihistaminics and mast cell stabilizer reduce
allergy and may prevent recurrence.
• Surgery in sinusitis is employed for mucocele,
osteomyelitis or chronicity unresponsive to medial
therapy. The aim of surgery is to establish effective
sinus drainage, by widening the ostium or removal
of the diseased mucosa to re-establish healthy
mucosal blanket. Endoscopic sinus surgery can
remove the anterior ethmoidal cells, the seat of
infection for spread to maxillary and frontal sinuses.
The posterior ethmoidal cells and even the sphenoid
sinuses can be approached. Patients with severe
allergy, recurrent polyps or immunocompromised
status have low success rate with endoscopic sinus
surgery. Major complications include CSF leak,
orbital haematoma, lacrimal duct obstruction,
bleeding from sphenopalatine arteries. Traditional
approach of antral wash/lavage and antrostomy are
done where functional endoscopic sinus surgery is
unavailable.
• External ethmoidectomy from Lynch incision or
external approach to frontal sinus from same incision
provide better view and complete removal of
diseased mucosa. The bicoronal flap is used in osteo-
plastic frontal sinus obliteration for chronic recurrent
frontal sinusitis. Sinus mucosa is removed and the
cavity obliterated using a fat graft and packing of
the frontal ducts with bone and muscle.
• The sphenoid sinus can be approached through its
anterior wall via intranasal endoscopic ethmoidec-
tomy or external ethmoidectomy.
36 Short Notes and Short Cases in ENT

10 Rhinitis

• Rhinitis is nasal hyperfunction and tissue inflam-


mation leading to nasal congestion, rhinorrhoea,
nasal obstruction, pruritus and sneezing. It can be
allergic and nonallergic. The former can be seasonal
or perennial. Infection is the most common cause of
acute rhinitis and allergy is the most common cause
of chronic rhinitis. Inhalants (Pollen, dander, mold,
dust, odour), food (fish, egg, nuts) and chemicals
(drugs) act as allergens to bind to IgE on mast cells
and basophils with release of mediators that
produce vasodilatation and immune inflammation
of allergy. Etiologic factors in non-allergic rhinitis are
infections (rhinovirus) structural, atrophic rhinitis,
emotions, temperature, hormonal, idiopathic (vaso-
motor rhinitis), systemic diseases (sarcoid, Horner’s
syndrome).
• Common structural abnormalities that can cause
rhinitis include deviated nasal septum, nasal valve
collapse, neoplasms (papilloma, angiofibroma),
polyps, nasal deformities).
• Atrophic rhinitis (Ozena) is associated with exces-
sive crusting and mucopurulent nasal discharge,
extremely foul odour, and epistaxis. It is idiopathic
without cure. Saline irrigation and topical antibiotics
may bring relief.
• Allergic rhinitis is treated with (1) antihistamines (2)
topical or systemic sympathomimetics (3) mast cell
stabilizers (cromolyn sodium) and topical or
systemic corticosteroids. Immunotherapy involves
injecting the offending antigen to the patient, which
decreases serum IgE, increases IgG (blocking anti-
body), decreases sensitivity of histamine releasing
cells and decreases responsiveness of lymphocytes.
• Rhinitis medicamentosa is drug-induced rhinitis,
most often associated with prolonged use of topical
decongestants that cause vascular atony after some
Rhinitis 37

time. Hence topical decongestants should not be


used for more than 3-5 days.
• Ipratropium bromide that antagonizes the effect of
acetylcholine at parasympathetically innervated
submucosal glands reduces nasal secretion in
rhinorrhoea and unlike topical vasoconstrictors does
not exhibit rebound on withdrawal.
• Topical corticosteroids effectively treat allergic and
non-allergic rhinitis without suppression of
hypothalamic pituitary adrenal axis. Injection of
inferior turbinate with triamcinolone can bring about
shrinkage; however intravascular injection by acci-
dent can cause orbital vessel spasm and blindness.
• Surgical therapy of rhinitis include septoplasty for
DNS, polypectomy and removal/reduction of
inferior turbinates. Cryosurgery of inferior turbinate
with vidian nerve sectioning can provide relief from
secretory aspects of vasomotor rhinitis.
• The danger triangle of face is from nasion to lateral
angle of mouth where from the venous drainage is
intracranial, i.e. from angular vein to inferior
ophthalmic vein and then into cavernous sinus.
• The lymphatics of nose and paranasal sinuses drain
anteriorly into submandibular or upper deep cervi-
cal nodes and posteriorly to retropharyngeal lymph
nodes.
• The nasal cycle is alternating air flow through the
nostrils due to reciprocal changes in resistance in
each nasal passage, the total resistance thus
remaining constant. This is due to autonomic
variation, primarily the adrenergic tone.
• The 4 views for PNS are (1) Water’s view for maxil-
lary and frontal sinuses (2) Caldwell view (PA view)
for frontal and ethmoid sinuses (3) Lateral view for
sphenoid sinus and posterior frontal sinus (4)
Submentovertex view for sphenoid sinuses. CT is best
for ethmoid sinuses and the osteomeatal complex.
• Nasal septum is made up anteriorly by quadrangular
septal cartilage, posteriorly by vomer and perpen-
dicular plate of ethmoid. Septal deviation commonly
involves the quadrangular cartilage to cause obstruc-
tion, meatal block and sinus infection.
• Inverted nasal papilloma is histologically benign but
clinically malignant, most common in the area of
38 Short Notes and Short Cases in ENT

ethmoid sinus and the opening of maxillary antrum.


It is a fleshy papillary exophytic growth having 13%
chance to convert to squamous carcinoma.
• Nasal biopsy is often used for diagnosis of
Wegener’s granulomatosis that affects upper respira-
tory passage and kidneys.
Nasal septal perforation—causes
• Following SMR or septal reconstruction
• Following septal absces
• Necrosis from cocaine sniffing
• Syphilis, leprosy, tuberculosis
• Wegener’s granulomatosis
• Relapsing polychondritis
• Neoplasm
• Idiopathic.
Otolaryngologic sequlae of allergies
Ear—Serous otitis media—due to eustachian block
Nose—Allergic rhinitis—type I hypersensitivity
Postnasal drip—increased nasal secretion
Nasal polyp—mucosal oedema
Throat—Angioedema—type I hypersensitivity.
Allergy testing
There are in vivo and in vitro techniques for identifying
specific allergens. These tests are also used to quantify
the strength of allergic reaction.
In vivo tests include patch, scratch, prick and
intradermal tests, in which a known quantity of sus-
pected allergen is introduced into the skin to react with
IgE on mast cells to produce wheal and flare. Larger the
wheal and flare, stronger is the allergic reaction.
In vitro tests include radio allergosorbent test (RAST)
and paper disk radioimmunosorbent test (PRIST). PRIST
is used to know total IgE level. RAST is used to measure
allergen-specific IgE antibodies rather than total serum
IgE.
11 Pharynx and Larynx

Anatomy
The pharynx is divided into nasopharynx (portion
above the margin of soft palate); the oropharynx (the
portion between the tonsillar pillars laterally, soft palate
superiorly and the epiglottis inferiorly) and the
hypopharynx (the portion from the base of the tongue
and epiglottis to the cricopharyngeus muscle inferiorly.
The eustachian tubes open posterolaterally to
nasopharynx and are surrounded by the cartilaginous
torus tubarius. Behind the eustachian opening are
mucosal recesses, the fossae of Rosenmuller. The adenoids
or pharyngeal tonsils hang from these fossae and the
posterosuperior nasopharyngeal wall. The hypo-
pharynx is surrounded by the three constrictor
muscles—superior, middle and inferior innervated by
IX and Xth cranial nerves. The hypopharyngeal space
extends superiorly into a recess between base of tongue
and epiglottis- the vallecula. The space extends inferiorly
into a recess on either side of larynx bordered medially
by aryepiglottic folds–the piriform sinus.
The larynx is at the level of C4-C6 vertebra. The
thyroid cartilage is shield shaped with superior and
inferior cornu that articulate with hyoid and cricoid
respectively. The cricoid cartilage is a complete ring,
articulating with thyroid and arytenoid cartilages. The
cricoid ring is narrow anteriorly and wide posteriorly.
It is attached anteriorly to thyroid cartilage by trico-
thyroid membrane. The paired arytenoid cartilages sit
on the posterior cricoid lamina in a saddle configuration
and articulate in a synovial joint that allows them to
glide, rotate and tilt. This synovial joint may dislocate,
ankylosed or inflamed. Most intrinsic muscles of larynx
attach to muscular process of arytenoid cartilage and
the true vocal folds attach to its vocal process. Arytenoid
action is responsible for opening and closing of glottis.
40 Short Notes and Short Cases in ENT

The paired aryepiglottic folds unite the epiglottis and


the arytenoids, corniculate and cuneiform cartilages.
Beneath the aryepiglottic folds are the false vocal cords,
the folds of mucous membrane. The true vocal cords
extend from vocal process of arytenoid cartilage to a
point in thyroid cartilage midway between its thyroid
notch and inferior border.
The intrinsic muscles of larynx move the vocal cord.
like adduction , abduction, tensor.
Adductors—Thyroarytenoid, lateral cricoarytenoid,
interarytenoid.
Abductor—Posterior cricoarytenoids.
Tensor—Cricothyroid.
All intrinsic muscles of larynx are supplied by
recurrent laryngeal nerve except for cricothyroid which
is supplied by superior laryngeal.
1. Hoarseness
Though in real terms hoarseness refers to laryngeal
dysfunction caused by abnormal vocal cord vibration,
in general it can reflect abnormality anywhere along the
vocal tract, from oral cavity to lungs. There are three
phases of speech—pulmonary, laryngeal and oral.
Pulmonary phase provides larynx with the column of
air, allowing vocal cords to vibrate at certain frequencies
to produce sound. The sound is finally modified in the
oral phase. Words are formed by the action of pharynx,
tongue, lips and teeth. Dysfunction in any of these
stages can cause voice change to be interpreted as
hoarseness by the patient.
When the speech abnormality is limited to tracheo
bronchial tree or lungs, patient has a weak, damped
voice. Oral stage abnormality may also lead to a muffled
or hot potato voice but true hoarseness from laryngeal
origin results in a rough raspy voice. The different
frequencies produced by vocal cord vibration depend
upon changes in length, breadth, elasticity and exten-
sion of the vocal cords.
Stridor indicates noise emanating from upper airway
during inspiration due to obstruction. It may coexist
with hoarseness when the obstruction occurs at the level
of vocal cords.
• Acute hoarseness is mainly due to laryngitis, voice
abuse or smoking whereas chronic hoarseness is due
Pharynx and Larynx 41

to polyp, vocal cord nodule, laryngeal papillo-


matosis, tumor, vocal cord palsy, or is functional.
Postnasal drip, GE reflux, altered laryngeal
architecture can lead to hoarseness.
• All patients of hoarseness be asked about presence
of coexistent sore throat, otalgia, dysphagia, odyno-
phagia, cough, haemoptysis, singing, etc.
• Thorough examination of nasal/oral cavity, neck
nodes and pneumatic otoscopy be done followed by
indirect laryngoscopy.
• The vocal cord is innervated by recurrent laryngeal
nerve, its fibers originate in nucleus ambiguous in
medulla and travel with vagus to exit through
jugular foramen. In the thorax vagus gives off the
recurrent laryngeals, the left one ascending up
looping around arch of aorta and the right around
subclavian artery to supply the vocal cords and
larynx inferiorly. The superior laryngeal nerve, a
branch of vagus innervates cricothyroid. Hence
vocal cord palsy can be due to tumor around jugular
foramen, neck, thyroid, bronctius and oesophagus
or in mediastinum. Thyroid tumors are most
common cause of bilateral vocal cord palsy.
Enlarged left atrium, aortic aneurysm, mediastinal
lymphadenopathy and fibrosing mediastinitis can
cause recurrent laryngeal palsy and hoarseness.
• In hysterical aphonia, the vocal cords paradoxically
abduct during speech but they have a normal cough.
• Laryngeal oedema can cause hoarseness. Though
mostly inflammatory noninflammatory causes
include obstruction to venous/lymphatic drainage
myxomatous tissue deposition, inhalants, anaphy-
laxis, etc. Laryngeal webs, cricoarytenoid disease
(rheumatoid arthritis) acromegaly, relapsing poly-
chondritis can all cause hoarseness. Vagal palsy due
to basal meningitis, polyneuritis, botulism, bulbar
poliomyelitis can all lead to hoarseness.
• Chronic simple laryngitis is mostly of viral origin
with dull red oedematous mucosa and responds to
voice rest, steam inhalation, NSAIDS and low dose
corticosteroids.
• Cancer of larynx is common to male in fifth or sixth
decade. Most laryngeal concers arise in the anterior
segment of vocal cord and hoarseness is an early
42 Short Notes and Short Cases in ENT

symptom. In cancer arising beyond vocal cord,


hoarseness occurs late but neck node involvement
occurs early. Ninety percent of laryngeal cancers can
be cured if detected at its early stage. Hence
hoarseness never be neglected in elderly.
• Laryngeal leukoplakia is a frequent cause of hoarse-
ness in smokers and has tendency for malignant
transformation.
• Vocal nodules also called Singer’s nodules tend to
arise bilaterally at the junction of anterior and
middle-thirds of true vocal cords, mostly due to vocal
abuse/over use. Early nodules are no more than
submucous haemorrhages and resolve with rest.
Long-term vocal abuse produce nodules that are
hard, white and fibrosed. Nodules are most common
cause of hoarseness in children. Only refractory
nodules not responding to voice rest need excision.
• Vocal cord polyps may be fusiform, pedunculated,
haemorrhagic or edematous (Reinkes’ polyp usually
occurs on one cord on anterior to middle third of true
vocal cords. Reinke’s edema or diffuse polyposis
commonly occur among older men with history of
smoking and vocal abuse. Surgical removal is often
necessary to ruleout cancer.
• Gastroesophageal reflux can cause hoarseness due
to laryngeal oedema. Similarly postnasal drip of
chronic rhinosinusitis can cause laryngeal inflam-
mation, oedema and hoarseness.
• Recurrent papillomatosis is the most common
benign neoplasm of larynx. They are though com-
mon in all ages, children are specially vulnerable.
HPV 6 and 11 are commonly responsible. They are
often thought to be related to hormonal changes and
regress after puberty; laser excision is curative.
• Vocal cord changes can be due to systemic disease
like acromegaly, hypothyroidism, and hyperthyroi-
dism. A patient with acromegaly may have low deep
voice. In hypothyroidism the voice has a flat low
pitch with dull coarse raspiness.
• Spasmodic dysphonia results from uncontrolled
contraction of laryngeal muscles. It is characterized
by strained or broken voice. A less common form of
spasmodic dysphonia is known as abductor spas-
modic dysphonia. It is characterized by intermittent
Pharynx and Larynx 43

breaks in phonation with severe breathlessness or


even complete aphonia. Botulinum toxin injection
into affected laryngeal muscles can bring relief, else
recurrent laryngeal ablation may be considered.
2. Laryngeal cancer
• Generally 67% of laryngeal cancers are glottic, 31%
supraglottic and 2% subglottic. Ninty-five per cent of
glottic cancers arise from true vocal cords. When diag-
nosed in 60% cases the disease is confined to larynx;
25% have regional metastasis and 15% have distant
metastasis. Ninty-four per cent of these cancers are
squamous cell carcinoma, 2-4% verucous carcinoma.
• Smoking and alcohol abuse are the most common
risk factors. Only less than 5% cancer larynx patients
have no smoking history. Less important risk factors
include oesophageal reflux, presence of laryngocele,
and history of uvenile papillomatosis.
• The larynx is divided into supraglottis and sub-
glottis. The supraglottis extends from tip of epiglottis
to the ventricular fold, the glottis extends from ventri-
cular fold to 1 cm below the vocal cord; while the
subglottis extends from here to inferior border of
cricoid cartilage.
• Supraglottis has rich lymphatic supply, hence nodal
metastasis is common. Glottis has limited lymphatic
drainage hence spread to neck nodes is less common
but hoarseness is an early symptom. Glottic cancers
are usually slow-growing and well-differentiated
with predictable ways of extension.
• Subglottic cancer is often silent, poorly differentiated
and with wide extension at diagnosis heralding
poor outcome.
• The quadrangular membrane above false cords and
the conus elasticus between the true cords and cricoid
cartilage act as barriers to spread of cancer. The
thyroid and cricoid cartilage are lateral to these
structures and once involved suggest poor prognosis.
Staging system for glottic cancer
T 1 Tumor limited to vocal cord/cords with normal
mobility.
T2 Tumor extending to supra/subglottis with limited
mobility.
44 Short Notes and Short Cases in ENT

T3 Tumor confined to larynx with vocal cord fixation.


T4 Tumor invading through laryngeal cartilage and/
or direct extralaryngeal spread.
• A nonworking vocal cord is fixed and demands
exclusion of cancer, either in/around the larynx or
on the pathway of vagus or recurrent laryngeal
nerve. Tumor invasion into thyroarytenoid muscle
or arytenoid cartilage leads to immobility of the cord.
There are many other causes of vocal cord paralysis
like traumatic, infective, congenital, idiopathic and
iatrogenic but cancer is to be ruled out first. Laryngeal
cancer with mobile cord is in stage T1 or T2 but with
fixed cord it is T3 which lowers 5 years survival from
80 to 95% to 30 to 50%.
• Reinke’s space is a potential space beneath the
epithelium of vocal folds. It contains loose connective
tissue and can transport cancer cells throughout the
cord, once through this space cancer can invade
thyroarytenoid muscle (vocalis muscle) causing cord
paralysis. The paraglottic space around the glottis
acts as a conduit for cancer to spread between all
divisions of larynx. Cartilage invasion spells poor
prognosis.
• Cricothyroid is the only extrinsic muscle of larynx
and is supplied by superior laryngeal nerve. It causes
adduction of vocal cord. All intrinsic muscles of
larynx are supplied by recurrent laryngeal. Hence
in recurrent laryngeal palsy, the cricothyroid
adducts the cord to midline. Bilateral recurrent
laryngeal palsy hence causes stridor and respiratory
obstruction demanding tracheostomy. When vagus
is paralysed intracranially, the cord assumes
intermediate or cadaveric position. With superior
laryngeal palsy, laryngeal sensation is lost and
threatens recurrent aspiration, often requiring
laryngectomy.
• Stage T1 and T2 disease, both supraglottis and
glottis respond well to surgery or radiotherapy with
cure rate of 75 to 90%. However, T3 disease has a
better salvage rate with surgery (50-80%) as
compared to 25% with radiation.
• Near total laryngectomy is a surgical procedure to
treat advanced stage disease by constructing a phy-
siologic mucosal shunt with the remaining arytenoid
to produce speech and requires a tracheostomy.
Pharynx and Larynx 45

• A laryngeal cancer patient has 5-10% chance of


developing a second primary cancer in the lung after
treatment of laryngeal disease; hence should be
followed up with chest radiograph.
• The pyriform sinus is the inferior extent of hypo-
pharynx just before it turns into the cervical
oesophagus. It is a funnel-shaped mucosa lined
sinus that invaginates between and around the
thyroid and cricoid cartilages. It opens during
swallowing to direct food into oesophagus. Cancer
of pyriform fossa is often silent. Medial wall pyriform
sinus cancers invade the larynx through paraglottic
space. They often present late as neck mass and
referred ear pain. Three-year survival is only 40%
with treatment.
• Anterior commissure glottic cancer spreads quickly
to thyroid cartilage. They are often understaged and
deep invasion diminishes effectiveness of radiation.
6. Tracheal tumors
Tracheal tumors are rare, representing 2% of all upper
airway tumors. Ninety per cent of them are malignant
in adults mostly squamous cell carcinoma and adenoid
cystic carcinoma. The benign lesions are—papilloma,
pleomorphic adenoma, granular cell tumor, fibrous
hystiocytoma, condroma, etc.

Squamous carcinoma
• Strongly associated with smoking
• Lymph node metastasis common
• Grows rapidly and aggressively
• Invades mediastinal structures.

Adenoid cystic carcinoma


• Not associated with smoking
• Lymph node invasion rare
• Slow indolent growth
• Displaces mediastinal structures.
• Secondary tracheal tumor is a malignant tumor that
involves or invades the trachea from an adjacent site,
e.g. laryngeal cancer, bronchial cancer, oesophageal
cancer, anaplastic thyroid cancer, etc.
• Ninety per cent of tracheal tumors in children are
benign-papilloma and haemangioma being most
common.
46 Short Notes and Short Cases in ENT

• Tracheal tumors present with cough, dyspnoea,


stridor, wheezing and haemoptysis. A patient with
recurrent bilateral pneumonia and no evidence of
chronic aspiration be considered for tracheal tumor
unless proved otherwise. Similarly patients of adult
onset asthma be evaluated for tracheal tumor.
• Respiratory papillomas are the most common benign
tumors of trachea. In the paediatric patient multiple
sites are involved throughout the laryngotracheo-
bronchial tree. But adult patients usually have single
descrete lesion. Endoscopic ablation is the treatment,
else CO2 laser vaporization may be effective. Most
malignant tracheal tumors bear a poor prognosis.
Surgical ablation with anastomosis of the free ends
can be attempted. Trachea is 10-11 cm long in adult
and upto 5-6 cm can be sacrificed. However tracheal
anastomosis carries risk of anastomotic dehiscence,
damage to recurrent laryngeal nerve, suture line
granuloma and restenosis. Postoperative radiation
is advised as squamous cell carcinoma and adenoid
cystic carcinoma are radiosensitive.
7. Laryngoscopy
Direct laryngoscopy is considered a surgical procedure
even though it does not require an external incision.
Fiberoptic laryngoscopy is performed through patient’s
nose which is sprayed with cocaine, that accomplishes
both anaesthesia and vasoconstriction. However,
biopsy, vocal cord injection or any surgical procedure
are better accomplished by direct laryngoscopy. Teflon
or fat is injected to vocal cord for medialization and
often boltulinum toxin is injected. General anaesthesia
with cocainization of larynx is required in direct
laryngoscopy. Cocaine decreases laryngocardial reflex,
minimises chances of laryngospasm after anaesthesia
wares off and is a vasoconstrictor preventing postbiopsy
bleeding.
Video laryngoscopy is the best way to assess
dynamic functions of larynx which can be aided by
stroboscope to document mucosal waves. With stro-
boscopy lesions such as very early carcinoma may be
seen as adynamic segment of vocal cord.
Pharynx and Larynx 47

Some other ancilliary tests used for evaluation of


laryngeal function include acoustic analysis, aero-
dynamic ability, electroglottography, electromyography,
lineradiography, etc.
48 Short Notes and Short Cases in ENT

12 Epistaxis

• Causes of epistaxis include (1) Local disorders—


trauma, infection foreign body, inflammatory reaction
to respiratory tract infections, chemical irritants.
(2) Neoplastic disorders of nose and PNS including
nasopharyngeal angiofibromas. (3) Systemic disor-
ders—hypertension, chronic renal failure. (4) Neuro-
logical diseases—leukaemia, myeloma, haemophilia,
aspirin/NSAID use. (5) Others—sudden pressure
and climatic changes.
• Kiesselbach’s plexus (also known as Little’s area) is
the rich anastomotic site for anterior and posterior
ethmoidal arteries, sphenopalatine and descending
palatine arteries and the superior labial branch of
facial artery. Ninety per cent of nose bleed occurs
from this site. However, arteriosclerotic nose bleed
as in hypertension causes posterior epistaxis.
• Genetic disorders predisposing to severe recurrent
nose bleed occurs in Osler-Weber-Rendu disease,
where mucosal telangiectasia is present throughout
the respiratory and GI tract; thus requiring repeated
laser coagulation of bleeding vessel or skin grafting
of nasal cavity.
• When source of bleeding is identified initial app-
roach be silver nitrate cauterization of bleeding site
under local anaesthesia; when bleeding is heavy
electrical suction cauterization may be more useful.
Deep and bilateral cauterization may cause cartilage
damage and septal perforation.
• When bleeding continues, anterior pack be given
using 6-feet long ½” wide vaseline gauze coated with
antibiotic ointment. A posterior pack is indicated
when bleeding is mainly into throat, posterior nose
bleed is visualized or anterior pack cannot be
correctly placed.
Epistaxis 49

• The posterior pack can be constructed from finely


rolled gauze which is placed in affected choana
through a catheter passed through affected nostril
to mouth and the gauze secured to it by 3 silk
sutures, 2 of which come out from nose and another
from mouth. Alternatively a 14 Foley catheter with
30 ml balloon can be used but is less effective. A
posterior pack can be left in place for 3-5 days.
• When patient continues to bleed through pack or on
removal of pack arterial ligation be considered. When
bleeding is from superior nose ligation of anterior
and posterior ethmoidal arteries or when from
inferior and posterior nose, ligation of internal
maxillary artery to stop haemorrhage from spheno-
palatine artery be effective.
50 Short Notes and Short Cases in ENT

13 Nasal Trauma

Examination and management of nasal trauma


It is best to anaesthetize and decongest the mucosa prior
to examination. The septum should be examined for
fracture, deviation, perforation, haematoma and muco-
sal tear. The nasal bones should be palpated for any
instability, motion or crepitus.
A septal haematoma has bluish or reddish colour,
often confused with deviated septum.
It should be drained by wide incision and a rubber
drainage with nose packed bilaterally with vasline
gauze.
When trauma to face is sufficient to fracture nasal
bones, it can also injure surrounding structures. The eye,
lacrimal system, paranasal sinuses, teeth and oral cavity
should be examined and possibility of CSF leak should
not be overlooked. Nasal bone fractures can be reduced
in first few hours after injury. After this oedema makes
accurate judgment of the degrees of deformity difficult.
Oedema generally subsides within 3-14 days after
which reduction can be attempted but before bony
healing starts. A closed reduction is reserved for simple
fracture with minimal displacement. A blunt instrument
is put in the anaesthetized nostril to raise the displaced
nasal bone followed by vaseline gauze packing inside
nose and a splint on the dorsum of the nose. However,
a fractured and displaced septum is best treated by open
reduction and septoplasty.
Paediatric nasal fractures be addressed quickly as
they heal fast and reduction becomes difficult after 5-
7 days. Injury to growth centres may cause disfigure-
ment. However, conservative management is key to
paediatric injuries. Airway obstruction, nasal deformity,
septal perforation, recurrent epistaxis and sinusitis may
complicate nasal fracture. Septal haematoma if not
drained may end in septal abscess, cartilage resorption
and saddle deformity. Septal abscess can lead to all
Nasal Trauma 51

septic intracranial complications due to drainage of


nasal veins to ophthalmic vein and sagittal sinus.

Causes of Upper Airway Obstruction


Nasopharyngeal obstruction
• Mediastinal disease
• Adenoid hypertrophy
• Aneurysm
• Nasopharyngeal tumors
• Lymphadenopathy
• Cysts (Thornwaldt’s encephalocele)
• Fibrosing mediastinitis.

Pharyngeal
• Parapharyngeal abscess
• Peritonsilar abscess (Quinsy)
• Ludwig’s angina
• Angioedema
• Tonsillitis.

Laryngeal and Tracheal


• Epiglottitis, croup
• Laryngeal tumor
• Subglottic stenosis
• Laryngomalacia
• Tracheal compression
• Vocal cord palsy
• Foreign bodies
• Laryngeal diphtheria
• Laryngeal oedema.

Angioedema can result in life-threatening upper air-


way obstruction. It is characterized by transient episode
of painless well-demarcated, nonpitting, asymmetric
oedema of the face, lips, tongue, mucous membrane, and
eyelids. About 20% of patients with angioedema have
severe airway obstruction. Causes range from idiopathic,
C1 esterage inhibitor deficiency, IgE mediated and drug
induced. Progressive dysponoea is the clue to diagnosis.

Signs of Upper Airway Obstruction


• Stridor—insipratory if obstruction is laryngeal
• Suprasternal retractions
• Drooling
52 Short Notes and Short Cases in ENT

• Subcutaneous emphysema
• Restlessness
• Abnormal voice—hoarseness in laryngeal disease.

Treatment of Upper Airway Obstruction


• Racemic epinephrine decreases mucosal oedema in
croup and laryngeal oedema.
• Corticosteroids also reduce oedema.
• Eighty percent helium + 20% oxygen mixture is good
as the lowered density of this mixture reduces airway
resistance to turbulent flow and thus reduces flow
resistive work.
• Endotracheal intubation is indicated to improve
respiratory toilet, assist ventilation, relieve obstruc-
tion and prevent aspiration.
• Nasotracheal intubation is simple and well-tolerated
in awake patient. It is advisable when there is injury
to cervical spine, or prolonged intubation is required.
Acute complications of endotracheal intubation
include acute pulmonary oedema—probably due to
sudden loss of highly negative intrathoracic pressure
during inspiration and positive pressure during
expiration. The subsequent increase in systemic venous
return and pulmonary hydrostatic pressure leads to
pulmonary oedema and aggravation of hypoxaemia
requiring diuretics and PEEP ventilation. Long-term
complications include laryngeal stenosis, recurrent
laryngeal nerve palsy, tracheoesophageal fistula.
Sinusitis occurs in patients who are nasotracheally
ventilated for longer periods.

Temporal Bone Trauma


Temporal bone fractures can be longitudinal or trans-
verse. Longitudinal fracture is more common (70-90%)
follows lataral blunt trauma to skull, and fracture runs
across middle ear along the long axis of petrous
pyramid. Transverse fracture occurs in 10-20%, follows
severe frontal or occipital trauma, begins at foramen
magnum extending across the temporal bone perpen–
dicular to long axis of petrous pyramid. It may involve
jugular foramen and almost always causes disruption
of the otic capsule with bleeding into middle ear, sensory
neural hearing loss and facial palsy (in 50%).
Nasal Trauma 53

Cholesteatoma may develop because of seeding of


mastoid cavity with squamous epithelium when
external auditory canal is fractured. Facial hypesthesia
occurs due to damage to gasserian ganglion on the
surface of petrous bone. Damage to abducent nerve in
Dorello’s canal causes diplopia. CSF otorrhoea, tympa-
nic membrane perforation and external auditory canal
laceration are seen in longitudinal fractures. CSF
rhinorrhoea occurs with transverse fractures.
Battle’s sign is postauricular ecchymosis that occurs
due to fracture across mastoid cortex. The raccoon sign is
periorbital ecchymosis. Both of them imply temporal bone
fracture. CT scan is the study of choice for temporal bone
fracture. Fractures of otic capsule do not heal easily and
lead to fistula formation with CSF leak and recurrent
meningitis.

Post-traumatic Vertigo
The causes include post concussive syndrome, concussive
injury to membranous labyrinth, cupulolithiasis, disruption
of labyrinth, perilymph fistula and endolymphatic hydrops.
Patients should be evaluated with ENG and audiogram.
Concussive injury to membranous labyrinth is the
most common form of post-traumatic vertigo. Vertigo is
positional with normal ENG. Fracture disrupting otic
capsule causes severe immediate vertigo with vomiting
and nystagmus. Perilymphatic fistula causes vertigo
and fluctuating/progressive sensory neural hearing
loss. Endolymphatic hydropes develops long after head
trauma causing hearing loss and tinnitus.
Conductive hearing loss (CHL) after temporal bone
trauma is usually temporary and is due to blood in
middle ear, oedema or TM perforation. Persistent CHL
is due to ossicular chain disruption. Sensory neural
hearing loss is due to otic capsule disruption, or
perilymphatic fistula. The latter is best diagnosed by
exploration.
54 Short Notes and Short Cases in ENT

14 Tonsils

Tonsillitis
Waldeyer’s ring is the lymphoid tissue located in naso-
pharynx and oropharynx consisting of palatine (faucial)
tonsils, pharyngeal tonsils (adenoids) lingual tonsils at
base of tongue and lateral “bands” on the lateral wall
of oropharynx. Passavant’s ridge is the bulge in the
posterior pharayngeal wall formed by the inter-
digitating fibers of superior constrictor muscle. The soft
palate abuts against it during swallowing and speech.
The tonsils and adenoids are lymphoid structures
and therefore play a role in immunology and host
defence. They produce secretory IgA that helps in
mucosal defense mechanism. The tonsils are supplied
from external carotid arteries by tonsilar and ascending
palatine branches of facial artery, ascending pharyngeal,
and lingual arteries. The anterior and posterior pillars
of tonsil are formed by palato-glossus and palato-
pharyngeus respectively.
Adenotonsilar disease is of childhood. Tonsils are
most active between 4-10 years of age at which time
adenotonsilar disease most often occurs. Involution
begins after puberty, but tonsillar disease frequently
occurs in adults as well.
Common bacterial pathogens of tonsillitis are beta
haemolytic streptococci and beta lactamase producing
organisms like bacteroides, staphylococci, Haemophilus,
and moraxella. Common viral pathogens include adeno-
virus, coxsackie virus, enteroviruses, parainfluenza
virus. EBV and RSV virus.
It is often difficult to distinguish bacterial from viral
tonsillitis. Hence culture is warranted. Though penicillin
is the drug of choice for streptococcal infection, betalac-
tamase- producing organisms respond best to clindamycin
or amoxycillin sulbactam/clavulanate preparation.
Quinsy is peritonsillar abscess or a loculation of pus
in the peritonsillar space that surrounds the tonsils. This
process develops as an infection in the peripheral
Tonsils 55

tonsillar crypt, penetrates through tonsillar capsule and


enters the peritonsillar space. Clinical features are dys-
phagia, odynophagia, trismus, and hot potato or muffled
voice but airway obstruction is uncommon. Examina-
tion reveals an inflamed oropharynx with inflamed
swollen tonsils and peritonsillar tissue displacing the
soft palate and uvula away from midline. Quinsy should
be drained by incision and tonsillectomy is performed
once there is resolution of infection.
Indications for tonsillectomy
Definite—(1) Recurrent tonsillitis, (2) Second quinsy, and
(3) Tonsillectomy for biopsy purposes.
Optional—Quinsy, recurrent or chronic tonsillitis, tonsi-
lolithiasis, cervical adenitis, halitosis, recurrent ill health.
Contraindication
1. Bleeding disorder
2. Recent infection (within 4 weeks)
Recent episode of tonsillitis increases risk of haemor-
rhage and surgery difficult.
Recent URT infection probably increases the risks of
pulmonary complications of general anaesthesisa.
3. Cleft palate
4. Oral contraceptives
5. During polio epidemic
Complication
Perioperative—Haemorrhage, trauma
Postoperative
• Immediate (24 hrs)—Haemorrhage, anaesthetic—
airway obstruction
• Intermedate (after 24 hrs to 10 days)—Haemorrhage,
haematoma and oedema of the uvula infection,
pulmomary complication, earach (referred pain)
• Late complication—Postoperative scarring, tonsillar
remnants
Velopharyngeal insufficiency may occur after
adenoidectomy due to incomplete closure of soft palate
against posterior pharyngeal wall with hypernasal
speech and nasopharyngeal regurgitation.
Indications for adenoidectomy
• Recurrent or chronic middle ear disease
• Sleep apnoca
• Obligate mouth breathing or snoring
• Recurrent or chronic adenoiditis
56 Short Notes and Short Cases in ENT

Tonsillectomy should be considered if there are (1)


7 tonsillitis attacks in 1 year (2) 5 infections per year for
2 consecutive years (3) 3 infections per year for 3
consecutive years or (4) 2 weeks missed from school due
to tonsillitis in 1 year.
Contraindications for tonsillectomy/adenoidectomy
• Cleft palate
• Blood dyscrasias
• Uncontrolled systemic disease
• Recent infection
• During polioepidemic
Complication of tonsillectomy/adenoidectomy
• Airway obstruction, haematoma and oedema of uvula.
• Bleeding (up to 7-10 days).
• Velopharyngeal insufficiency.
• Pulmonary oedema (especially after relief of obstruc-
tion).
• Infants are more vulnerable for difficulty in brea-
thing because of (1) Narrow airways that become
obstructed easily, (2) Less compliant lungs and
thorax, (3) Horizontal rib position yielding less inter-
costal excursion (4) Easy fatiguability of respiratory
muscles.

Stridor
• About 60% of stridor in children is localized to larynx,
15% to trachea and 5% to lungs. Expiratory stridor
is due to obstruction in distal bronchi whereas
biphasic stridor (both inspiratory and expiratory) is
due to obstruction immediately below the vocal cords.
• Definitive diagnosis of stridor generally requires
endoscopy of upper aerodigestive tract. Flexible
fiberoptic laryngoscopy and bronchoscopy are well-
tolerated in children.
• Most common cause of childhood airway obstruction
is croup or acute laryngotracheobronchitis, presen-
ting with barking cough and inspiratory or biphasic
stridor. The stridor is due to oedema of subglottic
space. Nebulized racemic epinephrine and oral/
parenteral steroid, supplemental O2 and fluids bring
quick improvement. Complication of croup include
pulmonary oedema, pneumonia and cardiac failure.
Tonsils 57

Epiglottis
• Epiglottitis occurs in children of 2-6 years with sore
throat, high fever, drooling of saliva, tachypnoea and
stridor. H. influenzae type B is the most common
causative agent. With appropriate IV antibiotics, the
epiglottis returns to normal size within 48 hours.
Epiglottitis Croup
Cause Bacterial Viral
Obstruction Supraglottic Subglottic
Drooling +++ +
Dysphagia Marked None
Posture Sitting Lying
Cough None Brassy, barking
Voice Muffled Hoarse
Course Shorter Longer
Onset Sudden (hours) Gradual (days)
Neoplastic disorders causing airway obstruction in
childhood
i. Haemangioma is the most common head and neck
neoplasm of childhood. When located in larynx,
it causes stridor.
ii. Juvenile nasopharyngeal angiofibroma is the most
common vascular mass in nasal cavity of children
and young adults. It is locally invasive, can lead
to gross facial deformity, can spread intracranially.
iii. Teratoma can occur in nasopharynx of children
with nasal obstruction.
iv. Lymphangiomas are soft compressible masses
appearing in children before 3 years of age. Airway
compromise may occur due to intraoral or
laryngeal extension.
v. Respiratory papillomatosis can occur in any part
of aerodigestive tract with respiratory obstruction,
hoarseness and even aphonia.
6. Salivary gland tumors
Benign Malignant
Pleomorphic adenoma Mucoepidermoid carcinoma
Monomorphic adenoma Adenocarcinoma
Warthin’s tumor Adenoid cystic carcinoma
Oncocytoma Epidermoid carcinoma
Acinic cell carcinoma

• Facial nerve palsy with a parotid tumor suggests the


mass to be malignant.
• A biopsy of parotid mass is contraindicated because
of (i) tumor spillage (ii) damage to facial nerve.
58 Short Notes and Short Cases in ENT

• The major salivary glands differ histologically. The


parotid gland is made up of basophilic serous cells,
which are arranged in acini or grape-like clusters.
The submandibular gland is a mixed gland contain-
ing both serum and mucous cells. The sublingual
gland contains primarily mucinous cells.
• Acinic cell carcinoma and malignant oncocytoma are
low grade malignancy.
• During parotid surgery identification of facial nerve
is most crucial. The nerve is located just inferior to
external auditory canal where it exits stylomastoid
foramen. It lies deep to retrofacial vein at the inferior
portion of the gland. The marginal mandibular
branch passes over the facial artery at the anterior
border of masseter.
• Recurrence of mixed tumor is not uncommon. It is
no more a discrete mass but multiplicity of nodules.
A recurrence may appear in previous scar,
subcutaneous tissue, superficial or deep parotid
parenchyma, facial nerve sheath, or perichondrium
of external meatus.
• Radiotherapy is indicated in parotid tumors in
following situations (1) Recurrence after surgery
(ii) Tumor involving or close to facial nerve (iii) High
grade tumors (iv) Lymph node metastasis (v) Extra-
parotid extension, (vi) T3-T4 (>4 cm in size).
• Frey’s syndrome is called gustatory sweating. It is
characterized by flushing and sweating of the skin
over surgical site. It occurs because of the post-
operative growth of preganglionic parasympathetic
to superficial sweat glands. Jacobsen’s neurectomy
(interruption of preganglionic parasympathetics to
parotid) relieves the symptoms.
15 Oral Medicine

Oral Lesions
• Saliva contains a variety of polypeptides and
glycoproteins that have antimicrobial activity. In
salivary hypofunction oral candidiasis and dental
caries are likely to occur.
• Medications are the most common cause of oral
dryness (xerostomia), e.g. TCA, antipsychotics,
belladona alkaloids, centrally-acting antihyperten-
sives like clonidine and antihistaminics. Xerostomia
develops when salivary flow is reduced at least by
50%.
• Aphthous ulcers are of common occurrence. Though
exact cause is unknown, stress, trauma, food allergy,
nutritional deficiency (B12, folate, iron), bacteria
(Streptococcus sanguis), HSV and immunologic abnor-
malities have been implicated.
• Aphthous ulcers can be major, minor and herpeti-
form. Minor ulcer is < 1 cm and is localized to freely
movable keratinized gingiva. It is white in center
with red border. Though extremely painful it resolves
spontaneously in 7-10 days. Major ulcers occur on
movable mucosa, tongue, soft palate and tonsilar
pillars, are more painful, of 1-3 cm in size.
Herpetiform ulcers are numerous and can coalesce
to form bigger ulcers which may leave a scar on
healing unlike major and minor ulcers.
• Medical treatment of aphthous stomatitis is oral
antibiotic, NSAID or immunosuppressants (local/
systemic steroids). Lactobacillus supplement is
beneficial. Doxycycline and tinidazole that take care
of chlamydia and anaerobes may be rewarding.
• Oral ulcerations also occur in systemic diseases like
Reiter’s, Behcet’s, pemphigus.
• Small creamy white curd-like lesions on tongue and
oral mucosa are features of thrush (candidiasis).
Antibiotic use, malnutrition, poor oral hygiene,
60 Short Notes and Short Cases in ENT

diabetic state or immunodeficiency need exclusion


so also corticosteroid use, HIV disease, neutropoenia,
etc.
• Desquamative gingivitis affects females after 30
years of age. It is characterized by diffuse erythe-
matous desquamation, ulcers and bullae. Associated
conditions include lichen planus, pemphigoid,
pemphigus vulgaris, dermatitis herpetiformis, and
drug reactions. Incisional biopsy and immuno-
fluorescent studies may be required for diagnosis.
• White lesions of mouth are usually benign but 5-10%
of oral malignancy present as white lesion. White
lesions can be keratotic or nonkeratotic.
Keratotic Nonkeratotic
Leukoplakia Acute candidiasis
Chronic candidiasis Aphthous stomatitis
Tertiary syphilis Vesicobullous disease
Nevoid white lesions Drug reaction
Lichen planus Pemphigus vulgaris
Psoriasis Bullous pemphigoid
SLE Benign mucous membrane
pemphigus
Verruca Erythema multiforme
Squamous carcinoma Secondary syphilis
Psoriasis
Desquamative gingivitis
Trauma—mechanical, thermal,
chemical

2 . Pigmented oral lesion

• Malignant melanoma
• Addison’s disease
• Peutz Jegher’s disease.
• Hairy leukoplakia occurs in HIV disease. They are
white painless lesions usually on lateral aspect of
tongue caused by EB virus.
• Torus mandibularae and torus palatinus are
incidental oral findings. In the former there is bone
enlargement at insertion of mylohyoid on mandible
on lingual surface. In latter, bone enlargement is
located on hard palate at midportion.
Oral Medicine 61

• Fordyce’s spots are small yellowish granular lesions


of normal sebaceous glands seen in upper lip,
anterior pillar and angle of mouth.
• Trench mouth is acute necrotizing ulcerative gingivitis
(Vincent’s angina) caused by anaerobes. The typical
lesions are punched out crater-like depressions in
the inter dental papillae and along the gingival
margins covered by a gray pseudomembrane.

3. Oral papilloma
Oral papillomas are the most common benign lesions
of oral cavity. HPV 6,11,16 and 18 are associated with
benign and malignant mucosal papillary neoplasms in
upper aerodigestive tract. Oral papillomas appear as
asymptomatic pink or white pedunculated lesions. Other
lesions caused by HPV include oral warts, verrucae
vulgaris which are identical to skin warts; condyloma
acuminata, a wart usually seen in anogenital region.

4. Angular cheilitis
Angular cheilitis is the presence of excoriated creases
at the commissures of lips. It is common to malnourished
or immunocompromised. Candida superinfection is
usual. Hence besides vitamin supplement, antifungal
local application (hamycin, mycostatin, clotrimazole)
bring improvement.

5. Median rhomboid glossitis


It is characterized by a deep red, smooth area in the
midline of the tongue of unknown origin. Some may
have underlying candidal infection.
Migratory glossitis (geographic tongue) is charac-
terized by patchy areas devoid of filiform papillae with
a raised yellow red border. Its cause is unknown. Pyrosis
or burning tongue is a nonspecific symptom and has
diverse causes from avitaminosis to malignancy.
Anaemia and hyperglycemia need exclusion. Local
disease like candidiasis, lichen planus, be excluded.
Black hairy tongue is due to elongation of filiform
papillae. The cause is unknown but it is associated with
antibiotic use, poor oral hygiene and nutritional
deficiency. Superficial desquamation by trichloracetic
acid may be helpful.
Oral lichen planus is an inflammatory disease where
the basal cell layer is destroyed by activated macropha-
62 Short Notes and Short Cases in ENT

ges. Women beyond 40 are usually affected. Emotional


stress may cause reactivation. The lesions are common
on buccal mucosa and may be reticular, plaque, atro-
phic, erosive and bullous. They may be asymptomatic,
painful or ulcerated. Biopsy is essential for diagnosis
and differentiation from white vesicobullous lesions and
to exclude malignancy which occurs in 1-2% cases.
Chronic discoid lupus erythematosus may have oral
lesions in 25% cases, clinically and histologically resem-
bling erosive lichen planus. They grow peripherally
with central healing. Biopsy shows perivascular
lymphocytic infiltrate in deeper submucosal layers.

6. Bad breath
Halitosis or bad breath is a fairly common symptom.
Bad breath may emanate from many regions of
aerodigestive tract, particularly nose and mouth. Ninety
per cent of cases of bad breath originate from oral cavity.
The presence of dental caries, plaque and food particles
lead to an overgrowth of bacteria resulting in foul breath.
Decreased salivary flow also hampers normal physio-
logic cleansing of mouth, allowing bacterial overgrowth.
Cryptic tonsils with retention of food particles and
subsequent bacterial growth can also cause bad breath.
Necrotising gingivostomatitis and oral ulcerating
neoplasms are also responsible.
Chronic sinusitis, atrophic rhinitis, tumors of nasal
cavity, infection of pharyngeal bursa (Tornwaldt’s
bursitis), the remnant of communication between
anterior notochord and nasopharynx can produce nasal
discharge and foul breath. Bronchiectasis, ulcerating
bronchial neoplasms and necrotising pneumonia can
impart foul odour to breath. Retention and degradation
of food in Zenker’s diverticulum can produce foul
breath. Bad breath can arise from systemic disorder like
hepatic failure, uraemia and diabetic ketoacidosis.
Halitosis may be feature of temporal lobe seizure.
Patients have delusions of halitosis in schizophrenia
and depression.

7. Throat pain
Acute pain
• Pharyngitis—viral (classic lymphonodular or
granular), bacterial
Oral Medicine 63

• Ludwig’s angia-cellulitis floor of mouth


• Infectious mononucleosis
• Ulcero-necrotic or acute tonsillitis
• Pharyngeal diphtheria
• Retropharyngeal abscess
• Peritonsillar abscess
• Epiglottitis
• Glossopharyngeal/superior laryngeal neuralgia.

Chronic Pain
• Chronic or persistent pharyngitis
• Chronic postnasal drip due to nasal allergy, chronic
sinusitis
• Gastroesophageal reflux
• Glossopharyngeal neuralgia
• Styalgia
• Acute/subacute thyroiditis
• Pharyngeal muscle tension
• Globus pharyngeus.

8. Styalgia
Chronic throat pain along the anatomic course of the
stylohyoid ligament is termed styalgia or Eagle syndrome.
It may be related to unrecognized elongation of styloid
process, a calcified stylohyoid ligament, or stylohyoid
muscle tendonitis. An elongated styloid process can
protrude into the tonsillar fossa and can compress
V,VII,IX,X cranial nerves. The patient may have chronic
sore throat, dysphagia or foreign body sensation in the
throat. Lateral neck radiography may yield the diag-
nosis. Surgical excision of styloid process is only
necessary when NSAIDs fail to control symptoms.

9. Globus Pharyngeus
It is a foreign body sensation in throat. It may be caused
by a variety of causes including mechanical, inflam-
matory, and neoplastic processes and gastroesophageal
reflux.
Nervous tension in many patients causes pharyn-
geal muscle tension and chronic throat pain reassurance
and often gentle muscle relaxant can bring improve-
ment.
64 Short Notes and Short Cases in ENT

16 Plastic Surgery in ENT

Rhinoplasty
It is repositioning and/or refinement of the nasal
skeleton and soft tissues to improve function, facial
aesthetics or both. Obstruction is the most commonly
addressed functional problem. Aesthetic concerns focus
on a dorsal nasal lump, poorly defined nasal tip or an
acquired deformity from trauma.

An aesthetic nose
• Nose length—middle one-third of facial length.
• Nose width—intercanthal distance of eyes or
one-fifth of facial width
o o
• Nose facial angle—30 -40
o o
• Nose labial angle—95 -100
• The sellion is a soft tissue landmark representing the
deepest point of the nasofrontal angle. The nasion is
a bony land mark at the nasofrontal suture. It is
slightly higher than sellion. The rhinion is a point
representing the bony- cartilaginous junction of the
nasal dorsum. The radix is the root of the nose.
• The three major anatomic areas that are addressed
in rhinoplasty are the septum, tip, and the dorsum.
Most incisions in rhinoplasty are situated around
the lower lateral cartilage.
• The three major tip support mechanisms are
(1) Cartilage of medial and lateral crura (2) Attach-
ments of medial crura to caudal septum (3) Attach-
ments of lateral crura to upper lateral cartilage.
• The two osteotomies done in rhinoplasty are medial
osteotomy to free nasal bones from septum and lateral
osteotomy to free nasal bones from maxilla. It is
important to free the nasal bones so that they can be
manipulated into their desired positions.
• The final results of rhinoplasty are only known six
months after operation. Although the bones heal in
six weeks, the soft tissue healing takes few months.
Plastic Surgery in ENT 65

Otoplasty
• The normal ear has an angle between the auricle and
o o
head of 25 -30 . The helical rim extends less than 20
mm from mastoid and a well-defined antihelical crus
is present. The best time to correct prominent ears
is between 4-6 years of age when ear growth is
essentially complete.
• When assessing an ear for otoplasty, the following
deficiencies should be anticipated (1) Poor anti-
helical folds (2) Deficient superior and inferior crus
around the fossa triangularis (3) Abnormal scapha
(4) Overdeveloped concha (5) Abnormal helix
definition and curvature.
• In lateral view the slope of the ear should approxi-
mate the slope of nasal dorsum. The ear should sit
slightly posterior to mid coronal plane on the head,
a distance said to be about one ear width from the
lateral orbital rim. Ear width is 60% of its height and
ear height is about 60 mm in adult.
• The six major goals of otoplasty are (1) Correct
protrusion of upper one-third of ear, (2) Allow helix
to be visible beyond the antihelix on AP view, (3) Give
the helix a smooth contour, (4) Achieve symmetry
between the 2 ears within 3 mm at any given point.
• To achieve above goals techniques are: (1) Removal
of postauricular skin, (2) Weakening of antihelical
cartilage, (3) Shaping the antihelical cartilage into
folds, (4) Reduction of conchal bowl if necessary.
• The head and neck syndromes having auricular
malformations are (1) Treacher-Collin’s syndrome,
(2) Hemifacial microsomia, (3) Goldenar syndrome.
• Microtia reconstruction surgery has 5 stages (1)
Auricular reconstruction, (2) Lobule transposition,
(3) Atresia repair, (4) Tragal reconstruction, (5) Auri-
cular elevation.

Rhytidectomy
Rhytidectomy is commonly known as face lift, aimed at
prevention of skin wrinkles in face. Most ageing changes
of skin occur in dermis in which the collagen, etlastin
and ground substances diminish with age. Normal skin
contains type I and type III collagen in ratio of 6:1.
Wrinkles occur in areas of muscle insertion and facial
animation. UVA is associated with actinic damage and
66 Short Notes and Short Cases in ENT

UVB with DNA damage and skin cancer. With actinic


damage dermis becomes thickened with increase in
elastin and immature type III collagen referred to as
elastosis. Common sources of UVA include sunlight and
fluorescent light.
The superficial musculoaponeurotic system (SMAS)
of face includes the platysma, rhisorius, triangularis,
and auricularis posterior muscles, connected to dermis
by fibrous septa. The nasolabial fold is formed by
insertion of muscles originating on the zygoma and the
insertion of the thinned SMAS. The greater auricular
nerve that supplies skin of ear is commonly damaged
during rhytidectomy. Rhytidectomy only addresses
lower two-third of face and neck line. Skin redundancy
in the lower face, laxity of neck, turkey gobbler deformity
and an obtuse cervicomental angle can all be improved.
Adjunctive procedures like forehead plasty, blepharo-
plasty and skin resurfacing can all be done for more
complete facial rejuvanation.
Rhytidectomy can be done in three planes-superficial
or subcutaneous plane, sub-SMAS plane and sub-
periosteal plane. Suction lipectomy has been used in
contouring the cheeks, and submandibular area.

Blepharoplasty
Blepharoplasty is intended to improve eyelid appearance
and function. Sometimes the upper eyelid becomes so
redundant that it may drape over upper eye lashes and
obstructs the patient’s view. Blepharoplasty addresses
four abnormalities—blepharochalasis, dermatochalasis,
pseudoherniation of fat and orbicularis muscle
hypertrophy. The Muller’s muscle which arises from
levator palpebrae and is inserted into superior border
of tarsal plate must not be damaged. It is supplied by
sympathetic through oculomotor nerve and maintains
the tone of upper eyelid. Injury to this muscle or the
levator leads to ptosis.
Blepharoplasty for lower lid employs skin flap
technique, skin-muscle flap technique, or transconjunc-
tival technique. Milia are the most frequent complica-
tions of blepharoplasty. These are white globular
nodules along suture track—effectively treated by pin-
point cautery or marsupialization with a knife.
Plastic Surgery in ENT 67

Skin resurfacing
Facial skin resurfacing may be considered for patients
with actinic keratoses , rhytids, pigmentary dyschromias,
superficial scarring, radiation dermatitis, acne vulgaris
and rosacea. Surgical techniques include chemical face
peels, dermabrasion, laser photothermolysis, and
injectable fillers. Peeling solutions are glycolic acid that
only burn the epidermis, trichloracetic acid that burns
through upper reticular dermis and Baker’s solution
that creates tissue damage through the midreticular
dermis.
Deep peel is indicated in actinic keratosis, photo-
aging, rhytids, lentignes, acne scarring, etc. Fillers,
commonly of collagen are injected under the skin to
elevate depressed scars or wrinkles. Rhinophyma is a
form of rosacea caused by sebaceous hyperplasia.
Common in elderly men, this hyperplasia leads to an
erythematous swollen nodular nose. CO2 laser treatment
is preferred for it.
68 Short Notes and Short Cases in ENT

17 Sleep Apnoea and


Snoring

Sleep apnoea is intermittent cessation of airflow during


sleep lasting for 10 seconds or longer. It can be central,
obstructive (OSA) or mixed. In central form the
respiratory center does not provide the necessary
respiratory motor signals whereas in obstructive form
respiratory effort is normal but there is upper airway
obstruction. Obstructive form is most common and the
typical symptoms include snoring, daytime somnolence,
morning headache, and restless sleep. Impotence and
hypertension are associated. Pickwickian syndrome
refers to obesity and hypoventilation. In obesity weight
of the neck, redundancy in soft palate and fullness in
base of tongue account for upper airway obstruction.
Retrognathia, low-lying hyoid bone and macro-
glossia also contribute to respiratory obstruction.
Chronic obstruction leads to hypoxaemia, pulmonary
hypertension and cor pulmonale. Diagnosis is based on
history and polysomnography. The latter measures EEG,
EMG, EOG (eye movements), pulse oxymetry, respiratory
effort and air movement at the nose and mouth.
Polysomnography can differentiate snoring without
OSA, pure OSA, central sleep apnoea and can indicate
the severity.
Severity of OSA is judged from respiratory distur-
bance index which is sum of the number of apnoeas
(cessation of airflow for >10 seconds) and hypopnoeas
(reduction of airflow by 50%) per hour. When RDI is >50
or oxygen saturation is <60% it is termed severe.
OSA in children is due to adenoid/tonsillar
enlargement. Craniofacial or orthodontic malformations,
nasopharyngeal cysts/encephaloceles and choanal
atresia can also be responsible.
Most obstructive symptoms occur during deeper
stages of sleep, i.e stage III,IV and REM sleep when
muscles are most relaxed with collapse of pharyngeal
wall causing the obstruction.
Sleep Apnoea and Snoring 69

Nonsurgical treatment of OSA include (1) Weight


reduction if obese, (2) Continuous positive airway
pressure, (3) Tongue retaining and mandibular posi-
tioning devices.
CPAP is maintained by a machine akin to ventilator.
Though it is 100% effective in relieving OSA, it is very
poorly tolerated. The retaining and positioning devices
open the airway by holding the tongue and mandible
forward during sleep.
Uvulopalatopharyngoplasty (UPPP) is the surgical
procedure for OSA where tonsils are removed, tonsillar
pillars are sewed together, posterior edge of soft palate
is removed to widen the oropharyngeal airway in the
anterior-superior and lateral dimension. Advancement
genioplasty combined with hyoid suspension where the
genial tubercle on mandible which attaches to genio-
glossus is advanced arteriorly and the hyoid bone is
suspended from mandible by permanent wires,
enlarges the airway at the base of the tongue. A third
procedure involving mandibular and maxillary advan-
cement by osteotomy address anatomic anomalies that
cause OSA and provide long-lasting relief.
Snoring without OSA can be addressed by laser-
assisted uvuloplasty or bovie-assisted uvuloplasty.
Even UPPP also stops snoring and relieves OSA.

Causes of Symptomatic Apnoea in Infants


Cardiac: PDA, arrhythmias, CHF
Congenital: laryngeal/tracheal web tracheal atresia epi-
gnathus, pharyngeal encephalocele, laryn-
gomalacia.
GI: GE reflux
Infection: RSV, botulism, meningitis, pertusis
Metabolic: Hypoglycaemia, electrolyte imbalance
Neurologic: Intracranial bleed, seizure, Arnold- Chiari
malformations
Pulmonary: Bronchopulmanory dysplasia.

Childhood disorders that can cause obstructive sleep apnoea


• Crouzon’s disease, Apert’s disease, Treacher-
Collin’s syndrome.
70 Short Notes and Short Cases in ENT

• Cystic fibrosis, neuromuscular disorders.


• Down syndrome.
• Hurler’s, Hunter’s syndrome.

When to suspect sleep apnoea in children.


• Irritable in morning.
• Night-time insomnia and enuresis.
• Developmental delay, failure to thrive.
• Cor pulmonale.
• Personality changes, learning problems.
• Morning headache.

Medical therapy for sleep apnoea in adults


• Weight loss.
• Acetazolamide.
• Almitrine.
• Avoid sleeping supine.
• Avoid alcohol, smoking, overeating.
18 Disorders of Smell
and Taste

Smell
The olfactory epithelium is along the superior turbinate,
cribiform plate and superior most portion of nasal
septum. It contains 4 main cell types. The ciliated
olfactory receptor is a bipolar neuron with a club-
shaped peripheral knob that bears sensory cilia. The
microvillar cell is the other sensory receptor. The
sustentacular cells are the supporting cells and the basal
cells act as stem cells for regeneration of sensory
receptor and supporting cells. The odorant produces
signal transduction by acting on a membrane-bound
receptor protein and the information passes to olfactory
bulb where it is processed and modified to travel along
olfactory tract to amygdala and pyriform cortex. When
the odorant is pungent, it can stimulate trigeminal nerve
endings throughout the nasal cavity.
Olfactory dysfunction can be classified into four
categories. Anosmia is complete loss of sense of smell,
dysosomia is distorted or perverted smell, hyposmia is
decreased sense of smell and hypersmia is increased
sensitivity to smell. Olfactory disorders can be conduc-
tive or sensory neural. Conductive disturbance is caused
by any process that prevents the odorant from reaching
the olfactory epithelium, e.g. developmental anomalies,
inflammation, neoplasia, trauma, etc. Mucosal oedema
due to rhinitis, DNS or PNS disease are the most
common causes of hyposmia or anosmia. Sensory neural
dysfunction is due to any factor involving olfactory
neuroepithelium or the central olfactory pathways. Most
common in this category is postviral dysfunction due to
partial or complete loss of olfactory epithelium (e.g.
influenza, HSV). Head trauma is the second most
common cause where the olfactory filaments are torn
across the cribiform plate. Many drugs like antineoplastic
agents, levodopa, cimetidine, antibiotics can impair smell.
72 Short Notes and Short Cases in ENT

Kallman’s syndrome and agenesis of neuroepithelium are


rare causes of anosmia. Anosmia may accompany
immune disorders (sarcoid, multiple sclerosis, Wegener’s)
avitaminosis, cerebral atherosclerosis, diabetes, Addison’s
disease, meningioma, craniopharyngioma, glioma, and
malignant CNS lesions.

Taste
The peripheral receptors for taste, the taste buds are
present in oral cavity (soft palate and tongue), pharynx
and cervical oesophagus. Of the four types of papillae
on tongue, only the fungiform, foliate and circumvallate
papillae are associated with taste but not the more
numerous filiform papillae. The fungiform papillae are
present on anterior two-third of tongue, are club-shaped
and red. The circumalate papillae are large circular
arranged in inverted V pattern at the junction of anterior
two-third and base of tongue. Taste fibers from
fungiform papillae travel in the lingual nerve, from soft
palate via greater petrosal nerve and from circumvalate
and foliate via glossopharyngeal nerve. Taste fibers from
these nerves join in the brain stem to form tractus
solitarius and are relayed to cortex via thalamus.
Gustatory dysfunction can be conductive or sensory
neural. Saliva helps to transport the tastant to the taste
buds and xerostomia is a major cause of taste loss.
Bacterial proliferation in oral cavity produces dysgeusia
or upleasant taste.

Gustatory dysfunction
Endocrine: Adrenal hypofunction/hyperplasia,
diabetes mellitus, hypothyroidism.
Oral, inflammatory: Oral candidiasis, gingivitis, perio-
dontitis, HSV pemphigus, Sjögren’s
syndrome
Neurologic: Bell’s palsy (chorda tympani), head
trauma, dysautonomia, multiple
sclerosis
Nutritional: B3 (niacin) deficiency, CRF, cirrhosis
Tumors: Base of skull, oral cavity
Drugs: ACE inhibitors
Otolaryngeal
19 Manifestations of
HIV Diseases

1. External Ear
Kaposi’s sarcoma, pneumocystis infection, otomycosis
are common to HIV cases. Kaposi’s sarcoma is raised
reddish blue nodule pneumocystis infection appears as
subcutaneous or cutaneous lesion or large antral polyp.
Biopsy shows large multiloculated cysts.

2. Internal Ear
Hearing loss is common in HIV disease, either due to
meningitis caused by bacteria, fungi and viruses or direct
damage of HIV to 8th cranial nerve. Many drugs used
to control infections in HIV disease are ototoxic.

3. Sinusitis
Unusual pathogens like Cryptococcus neoformans,
Alternaria alternata and Acanthameba castellani cause
sinusitis in HIV disease in addition to normal agents.
Giant herpetic nasal ulcers are also known to occur.
Kaposi’s sarcoma and non-Hodgkin’s lymphoma are
also known to occur in nasal vestibule, septum,
nasapharynx and sinuses in AIDS patients.

4. Oral Cavity
Kaposi’s sarcoma, candidiasis, HSV, hairy leukoplakia.

5. Larynx
Kaposi’s sarcoma, severe fatal epiglottitis.

6. Salivary Glands
Cystic parotid enlargement caused by benign lympho-
epithelial lesions.

7. Cervical
Lymphadenopathy due to nonspecific follicular
hyperplasia non-Hodgkin’s lymphoma or mycobacterial
infections.
74 Short Notes and Short Cases in ENT

20 Geriatric
Otolaryngology

• Dysphonia and dysarthria are the two speech


disorders of elderly. Dysphonia results from atrophy
of vocal folds, and laryngeal muscles, metaplasia of
false vocal cords, and reduced mobility of crico-
arytenoids and cricothyroid joints. Dysarhria is due
to cerebrovascular accidents, Parkinson’s disease
and Alzheimer‘s disease.
• Sensory neural hearing loss is due to atherosclerosis
of vertebrobasilar vessels leading to atrophy of organ
of corti in basal turns of cochlea (hence high
frequency loss). Four forms of presbycusis are known
(1) high frequency loss (2) progressive flat loss due
to some physical chemical process in middle ear (3)
high frequency loss with severly affected speech
discrimination (4) straight line descending curve,
attributed to disorder of mechanic of cochlear duct.
• Tinnitus in elderly is very disabling but may be
related to presbycusis or noise induced hearing loss.

Noise-induced Hearing Loss


Sensory hairy cells degeneration may occur on prolon-
ged exposure to loud noise. The initial maximum
hearing loss occurs at 4000 Hz. As damage proceeds,
the loss involves lower and higher frequencies.
Short Cases
76 Short Notes and Short Cases in ENT
Short Cases 77

Case No. 1

A 35-year old male is complaining of recurrent attacks


of cold, headache and intermittent epistaxis. Exami-
nation reveals S-shaped deformity of nasal septum.

Symptoms of deviated nasal septum


DNS is a common problem to Europeans, primarily
males. Though many of us have varying amount of
septal deviations, only few have symptoms like
• Nasal obstruction which may be unilateral, or
bilateral; continuous or intermittent.
• Dryness in mouth and pharynx.
• Recurrent attacks of cold, headache and facial pain.
• Epistaxis.
• Deformity of nasal tip.
• Anosmia and rarely sleep apnoea.
• Pain of anterior ethmoidal nerve compression.
Constituents of nasal septum
Nasal septum has bony and cartilaginous parts. Bony
septum is formed by:
• Posteroinferiorly by vomer.
• Posterosuperiorly by perpendicular plate of ethmoid.
• Nasal spine of frontal bone.
• Rostrum of sphenoid between vomer and ethmoidal
plate.
• Nasal crest of two maxillae and palatine bones.
The cartilaginous part is formed by quadrilateral
cartilage.
Blood supply of septum
• Main supply is from long sphenopalatine branch of
internal maxilary artery.
• Anterior and posterior ethmoidal arteries.
• Terminal branches of greater palatine artery.
• Septal branches of superior labial artery.
Sinuses/ducts opening into various nasal meatus
1. Inferior meathus
• Nasolacrimal duct.
2. Middle meathus:
• Anterior ethmoidal cells and frontal sinus into
anterior part of meatus.
78 Short Notes and Short Cases in ENT

• Middle ethmoidal cells open into hiatus semi-


lunaris.
• Maxillary sinus opens into posterior part of
hiatus semilunaris.
3. Superior meatus
• Posterior ethmoidal cells anteriorly.
• Spheroid sinus posteriorly to sphenoethmoidal
recess.
Function of paranasal sinuses
No definite function of paranasal sinuses is known, but
they are thought to serve following functions:
• Warning and moistening of inspired air.
• Resonator to laryngeal voice.
• Temperature buffers to eyes and brain.
• Reduce weight to facial bones, thus helping in
balance of head.
• Serve as mucosa donar e.g. maxillary sinus mucosa
to nasal cavity in atrophic rhinitis.
Treatment of deviated nasal septum
• Submucosal resection (SMR).
• Septoplasty.
• Septorhinoplasty.
Indications for SMR of septum
1. DNS producing nasal obstruction, epistaxis.
2. DNS as a predisposing factor for sinusitis, recurrent
colds, poor development of teeth and mouth.
3. DNS preventing access for removal of polyp or
ethmoidectomy.
4. To gain access for other intranasal operations like
transsphenoidal hypophysectomy, vidian neurec-
tomy.
5. For taking septal cartilage for graft.
6. To reduce roominess in unilateral atrophic rhintis.
Complications of SMR and its contraindications
Complications Contraindications
Haemorrhage age<18 Years
Perforation Elderly
Flapping septum Acute infection in nose
Depression of septum Specific disease (TB, Syphilis)
Drooping of the tip
Adhesion between septum and turbinates
Short Cases 79

Advantage of septoplasty over SMR


1. Septoplasty can be undertaken in children but not
SMR.
2. Since there is minimal resection of septum drooping
of nasal tip or collumella recession is not a problem.
3. Flapping of septum and perforation does not occur.
4. Revision surgery, if needed, is easy.
Causes of perforation of nasal septum
• Trauma due to surgery, tight nasal packaging, septal
necrosis.
• Syphilis, midline garnuloma, atrophic rhinitis, lupus
vulgaris.
• Tumors of septum-like Chondrosarcoma.
• Idiopathic.
What is Little’s area?
This is the anterior inferior part of nasal septum, the
commonest site for epistaxis. This area is highly
vasucular due to rich anastomosis of blood vessels
forming the Kieselbach’s plexus.
What is Woodruff’s area?
This area is near the posterior end of inferior turbinate.
It has a rich venous plexus and serves as another
common site for nose bleed.
Management of nose bleed
1. Pinching of nose and ice application can stop bleed-
ing in majority else adrenaline drops or spray (in
non-hypertensives), or cocaine 4% spray is helpful.
2. Cauterization of bleeeding point by dithermy/silver
nitrate.
3. Anterior nasal packing. Five percent cases bleed
from posterior nasal cavity and need a posterior
nasal pack in addition.
How to proceed with anterior nasal packing?
Half an inch wide and several feet long iodoform gauze
lubricated with xylocaine ointment and vaseline is
placed along floor and then valt of nose. The packing
is removed after 48 hours. If kept for longer period, the
pack may cause nasal ulceration.
Other methods to control nose bleed
1. Balloon tamponade.
2. Arterial ligation i.e.internal maxillary artery ligation
in sphenopalatine fossa or anterior ethmoidal artery
ligation through periorbital incision.
80 Short Notes and Short Cases in ENT

Case No.2

A young college-going boy has presented with long


history of sneezing—more so in morning and watering
from eyes. Examination reveals pale nasal mucosa with
yellowish boggy masses in both nostrils.

What is your diagnosis?


A case of allergic rhinitis with polyposis.
What is nasal allergy?
Nasal allergy occurs as a result of altered reactivity of
nasal mucosa to allergens. The allergens in contacts
with nasal mucosa cause mast cell degranulation,
release histamine and other kinds that produce vaso-
dilation, mucosal swelling and watery nasal discharge.
The allergy can be seasonal or perennial. Seasonal
allergy is due to inhalants like pollen of trees, grasses
and weeds. Perennial allergy is due to house dust,
smoke, spores and fungi, perfumes, food (egg, fish) and
drugs.
What is vasomotor rhintis?
It is a non-infective condition resulting from autonomic
disturbances as during emotional disturbances, preg-
nancy, menstruation, extremes of climate, antidepres-
sant and antihypertensive drugs. Conjunctiva is free, but
nasal congestion and watery discharge with hyper-
trophy of inferior turbinate are marked. Troublesome
nasal discharge can be controlled by vidian neurectomy.
What is common cold?
Common cold is viral rhinitis caused by numerous
stereotypes of adenoviruses and rhinoviruses without
production of any immunity, so that individual remains
sensitive throughout life.
Difference between viral rhinitis and nasal allergy
In viral rhinitis nasal mucosa is red and edematous with
watery discharge but in nasal allergy mucosa is pale or
violaceous (due to venous congestion).
Treatment approach in allergic rhinitis
• Nasal decongestants (phenylephire, xylometazoline).
• Antihistamines like astemizole, clemastine, cetirizine
and terfenadrine.
Short Cases 81

• Nasal corticosteroid sprays like flunisolide or


beclomethasone.
• Intranasal steroid may strink the polyp and
oedematous mucosa.
• Identification of allergen and desensitization.
Newer methods for prevention of common cold
Though no vaccine is available, research is continuing
to prevent the disease by developing antibody to cell
proteins that serve as attachment site for the viruses or
administering proteins of similar structure to attach to
viruse for elimination.
What is nasal polyp?
Polyp is a pedunculated hypertrophied edematous nasal
mucosa. Polyp commonly arise from ethmoid labyrinth.
Some may arise from maxillary antum, to project into
nasopharynx-called antrochoanal polyp.
Differential diagnosis of nasal polyp
• Hypertrophied inferior turbinate
• Rhinosporidosis
• Angiofibroma of septum
• Papilloma from middle meatus
• Harmartoma
• Nasopharyngeal angiofibroma.
How to differentiate hypertrophied turbinate from polyp?
Turbinate is pink in colour, sensitive to touch, with a
firm feel. A probe cannot be passed around turbinate.
Polyp is pale, soft, intensive with a peduncle, so that
probe can be passed around.
What is rhinosporidiosis?
It is fungal infection (R. seberi), producing a bleeding
polypoidal mass of strawberry appearance arising from
septum.
What is rhinitis medicamentosa?
It is reflex nasal congestion after prolonged use of nasal
vasoconstrictors. Nasal corticosteroid drops/sprays or
oral prednisone are helpful. Complete stoppage of
vasoconstrictor drops is essential.
What is atrophic rhinitis?
Atrophic rhinitis is a chronic inflammatory condition
of nose, characterized by atrophic changes in nasal
82 Short Notes and Short Cases in ENT

mucosa and the turbinates. It can be primary or secon-


dary.
Pathology of atrophic rhinitis
There is atrophy of ciliated columnar epithelium with
metaplasia to stratified squamous type. Mucus glands
and globlet cells are fewer and there is round cell
infiltration of subucosa. Endarteritis causes bone loss.
Nasal secretion is thick and viscid with crust formation
and secondary infection causing fetor.
Clinical features of allergic rhinitis:
• Dryness of nose
• Nasal obstruction
• Epistaxis
• More roomy nasal cavity with shrivelled up turbi-
nates, red atrophic nasal mucosa filled with crust.
Medical management of atrophic rhintis:
• Alkaline douching of nose
• 25% anhydrous glucose in glycerine
• Local application of estrogen
• Placental extracts
• Vitamin supplement.
Surgical therapy of strophic rhinitis
• Endonasal microplasty to narrow internal dimen-
sions of nose
• Transplantation of Stensen’s duct into maxillary
sinus
• Stellate ganglion block
• Nasal submucosal placental implantation.
Role of ascorbic acid in common cold prevention
Megadose Vit. C has been advocated for prevention of
common cold and cancer. Randomized clinical trials
have not conclusively proved about its efficacy in such
situations.
Short Cases 83

Case No.3

A 30-year old patient had presented with headache


more so in morning, mild fever and mucopurulent dis-
charge from nose. Examination reveals boggy swelling
of nasal mucosa with mucopus in middle meatus
posteriorly

What is your diagnosis?


A case of acute sinusitis.
Name the paranasal sinuses
• Maxillary
• Frontal
• Ethmoidal
• Sphenoidal
Which form of sinusitis is common and why?
Maxillary sinusitis is common because it is the largest
of paranasal sinuses and its ostia into nose are
superiorly placed, with incomplete evacuation.
Predisposing factors for sinusitis
• Nasal allergy
• Common cold
• Deviated nasal septum
• Dental infection (maxillary sinusitis)
• Adenoids and infected tonsils
• Congenital defective ciliary transport (Kartargener’s
syndrome).
Bacterial agents of sinusitis
• Pneumococcus
• Streptococcus
• H. influenzae
• S. aureus
• Moraxella catarrhalis.
Locations of pain in sinusitis
Maxillary sinus—In the cheeks below the eyes.
Frontal sinus—In the forehead.
Ethmoidal sinus—Over nasal bridge, inner aspect of
eyes.
Sphenoidal sinus—Behind the eyes and vertex.
84 Short Notes and Short Cases in ENT

X-ray examination for sinuses


1. Cald Well view for frontal sinus, where patient’s
forehead and tip of nose are kept in contact with the
film.
2. Water’s view (occipito-mental view) for maxillary
sinus, where nose and chin touch the film with
mouth open.
3. Lateral view for sphenoid.
4. Submento-vertebral for ethmoid.
Radiological evidences if sinusitis
• Sinus opacification without bone destruction.
• Air fluid level (in upright films).
• CT Scanning helps in better delineation of inflam-
matory changes, bone destruction and osteomeatal
block.
• When malignancy is suspected because of naso-
antral mass, unilateral cranial neuropathy MRI with
gadolinium contrast helps to distinguish a tumor
from inflammation and inspissated mucus.
Complications of sinusitis
• Local complications like osteomyelitis and mucocele.
• Intracranial complications in the form of cavernous
sinus thrombosis, epidural abcess and meningitis.
• Sixth nerve palsy in sphenoidal sinusitis as the nerve
courses just lateral to the sinus.
• Oro-antral fistula in chronic maxillary sinusitis.
• Repeated attacks end in chronic polypoidal changes
and atrophic sinusitis.
Role of ultrasound in sinusitis
A scan is helpful in identifying thick edematous sinus.
Mucosa from polyps and free fluid within the sinus.
Treatment of sinusitis
• Oral decongestant like pseudoephedrine and nasal
decongestant sprays/drops.
• A 2-week course of antibiotics like amoxycillus,
cephalexin.
• Maxillary antrum puncture for relief of pain and
culture of drained fluid.
• Endoscopic opening of osteomeatal complex or
intranasal antrostomy, when conservative treatment
fails.
• Ethmoidectomy for chronic ethmoidal infection.
Short Cases 85

What is Cold Well Luc operation and its indications?


It is opening of anterolateral wall of maxillary sinus
through canine fossa. Indications are:
• Chronic maxillary sinusitis
• Recurrent antrochoanal polyp
• Surgery of dental and dentigerous cyst.
What is mucocoele?
Mucocele is a cystic swelling commonly affecting
frontal and ethomoidal sinuses. The swelling contains
tenacious mucus with thinning of sinus wall and
expansion of the sinus. The cyst may get infected. It may
cause downward and outward displacement of eye.
Frontoethmoidectomy is the treatment of choice.
Diseases simulating chronic sinusitis
• Antrochoanal polyp
• Tumor from sinus.
Is cytology of antral fluid mandatory?
Yes, cytology may show shred malignant cells, as in
lymphoma involving maxillary sinus in AIDS patient
masquerading as sinusitis.
86 Short Notes and Short Cases in ENT

Case No.4

A case has been brought by a mother with acute pain in


the ears, cough and coryza. Examination reveals con-
ductive deafness and hypomobile ingested tympanic
membranes

What is diagnosis?
Acute suppurative otitis media (ASOM)
Pathogens of ASOM
• Streptococcus pneumonae
• Haemophilus influenzae
• Gram-ve enteric bacilli (in newborns).
Predisposing factors for ASOM
• Viral upper respitatory infections
• Eustachian catarrh
• Adenoids.
What is serous otitis media?
Auditory tube block for long period leads to negative
pressure within middle ear and serous transudates
especially in children, in whom the eustachian tubes are
short, narrow and straight. The tympanic membranes
is dull and hypomobile, occasionally accompanied by
air bubble in middle ear and conductive deafness.
What is retracted tympanic membrane (RTM)?
Absorption of air from middle ear cavity due to tubal
block leads to falling back of tympanic membrane
towards middle ear cavity. Patients complain of fullness
in ear and conductive deafness. Examination reveals
retracted tympanic membrane with prominent handle
of malleus and hypomobility of tympanic membrane.
Management of RTM
• Patient to be advised to swallow, yawn, do valslva
• Systemic and intranasal decongestants
• Adenoidectomy
• Air travel, rapid altitudinal changes and underwater
diving be avoided.
• Steroid treatment of nasal allergy
• Ventilatory tube placement (grumet) to equalize
pressure, when other methods fail.
Short Cases 87

What is barotrauma?
Individuals with auditory tube dysfunction due to
congenital narrowness or mucosal oedema may be
unable to equalize the barometric stress, exerted on the
middle ear by air travel, rapid altiudinal change or
underwater diving. During aeroplane descent, the
auditory tubes collapse causing acute pain in the ears.
Treatment of barotrauma
• Advice to patient to swallow, yawn and autoinflate
during descent.
• Systemic decongestant to be taken several hours
before air travel or nasal decongestant spray 1hour
before air travel.
• Infants be given bottle to suck during descent.
• Myringotomy if pain is distressing.
• Insertion of ventilating tubes, it patient has to
undertake frequent air travel.
Aural complications of underwater diving
• Barotrauma
• Haemotympanum
• Perilymphatic fistulization
• Labyrinthine dysfunction
Complications of ASOM
• Acute mastoiditis, meningitis
• Middle ear empyema
• Tympanic membrane rupture.
Treatment of ASOM
• Antibiotic like amoxycillin or cephalexin
• Nasal decongestants
• Analgesics and anti-inflammatory drugs
• Tympanocentesis to obtain pus sample for culture
(fungi, gram-ve organisms)
• Myringotomy to relieve acute pain.
What are the stages of acute mastoiditis?
At the initial stage, there is clouding of air cells, but the
bony partition between them is intact. Continued
infective process leads to thickening of mucoperiosteum,
decalcification of wall of air cells and many of them
coalesce to form large cavities. When the disease
becomes chronic, air cells are lost and mastoid becomes
sclerotic.
88 Short Notes and Short Cases in ENT

What is Gradenigo’s syndrome?


Petrous apicitis, a consequence to mastoiditis, causes
oedema of 6th nerve in Dorello’s canal. Hence, there is
sixth nerve palsy along with otorrhoe.
What is mastoid antrum?
Mastoid autrum is an air chamber in temporal bone. It
communicates anteriorly with tympanic cavity through
the aditus. Posteriorly, it communicates with mastoid
air cells.
What is Macewen’s triangle?
Macewen’s triangle is the surface anatomy of mastoid
antrum.. It is bounded above by posterior root of
zygoma, anteriorly by the posteriosuperior canal wall.
Behind, the triangle is completed by a line, which is
tangential to the posterior canal wall below and cuts
posterior root of zygoma above.
What are the air cell groups in mastoid?
1. Periantral cells.
2. Tip cells—superficial and deep.
3. Perisinus cells around sigmoid sinus.
4. Perilabyrinthine cells.
5. Retrofacial cells.
6. Petrosal cells.
7. Hypotymapnic cells.
8. Zygomatic cells.
Function of mastoid cellular system
Exact function of air cell is not known, but they may
serve the following:
• Air reservoir for the middle ear cavity.
• Insulate the labyrinth from temperature variations.
• May function as resonator.
What is Trautmann’s triangle?
The triangle lies behind the antrum, bonded by bony
labyrinth anteriorly, sigmoid sinus posteriorly, and
superior petrosal sinus superiorly. Infection can pass
to posterior cranial fossa through this triangle.
Short Cases 89

Case No. 5

A 10-year old boy is having intermittent mucopurulent


bilateral aural discharge for past 6 months. Examina-
tion reveals central tympanic membrane perforations
with external otitis and conductive deafness.

What is your diagnosis?


A case of chronic suppurative otitis media (CSOM).
Predisposing factors for CSOM
• Adenoids
• Deviated nasal septum
• Allergic rhinitis and polyps
• Nasopharyngeal tumor
• Milk regurgitation into nasopharynx due to
improper position during feeding of small babies
• Tymapnic membrane perforation secondary to
trauma, noise.
Classification of CSOM
• Tubotympanic type
• Attico-antral type.
Bacterial agents of CSOM
• P. aeruginosa
• Proteus species
• Staphylococcus aureus
• Anaerobes.
Significance of aural polyp
Repeated infection of middle ear leads to mucosal
hyperplasia and polyp formation, especially from
chronically infected sclerotic mastoids.
What is blue drum?
It is a non-suppurative form of middle ear disease with
blue gray discoloration of drum head and cloudiness
of mastoid in X-ray caused by accumulation of dark
brown slimy sludge in the middle ear mastoid process.
There is a vascular granulation tissue containing
cholesterol crystals, blood pigments and giants cells.
Blood cholesterol is usually high.
Treatment of CSOM
• Treatment of underlying cause i.e. disease of nose,
nasopharynx and PNS
90 Short Notes and Short Cases in ENT

• Daily aural toilet


• Myringoplasty.
Prerequisites of myringoplasty
• Ear should be dry for at least 6 weeks prior to
procedure
• Eustachian tube should be patent
• No focus of infection in nose and throat.
Advantages of myringoplasty
• Prevents further infection
• Improves hearing
• Prevents tympanosclerosis
• Enables proper fitting of hearing aid
• Enables person to be recruited to professions like
diving, military.
Graft materials for myringoplasty
• Temporalis fascia
• Tragal perichondrium
• Homograft tympanic membrane.
Why attic perforation is dangerous?
Attic perforation is accompanied by cholesteatoma
formation.
What is Cholesteatoma?
Cholesteatoma is a squamous epithelium lined sac,
filled with desquamated keratin, with a tendency for
chronic infection and bone erosion.
Complications of CSOM
Meningeal Nonmeningeal
Extradural abscess Mastoiditis
Perisinus abscess Petrositis
Lateral sinus thrombosis Facial nerve palsy
Otitic hydrocephalus Labyrinthitis
Meningitis Retropharyngeal abcess
Subdural abscess
Brain abscess.

Clinical tests for diagnosis of lateral sinus thrombosis


When one lateral sinus is thrombosed digital compres-
sion of opposite side internal jugular vein produces
dilatation of retinal veins and rise in intracranial
pressure.
Short Cases 91

What is otitic hydrocephalus?


Marked rise in cerebal venous pressure due to bilateral
lateral sinus thrombosis leads to increased CSF pressure
and its reduced absorption from the arachnoid
granulation.
Location of brain abscess in CSOM
The sites of parenchymal brain abscess are temporal
lobe and cerebellum. They most commonly evolve from
retrograde thrombophlebitis adjacent to an epidermal
abscess. The predominant causative organisms are
S. aureus, S. pyogenes and S. pneumonae.
Facial palsy and otitis media
Facial palsy may be associated with either acute or
chronic otitis media. In acute setting, it is due to
inflammation of the nerve, in its middle ear segment, due
to bacterial neurotoxins. In CSOM, paralysis evolves
slowly due to pressure on the nerve by Cholesteatoma.
The prognosis in these patients is less favorable than
for facial palsy associated with acute otitis media.
Treatment of cholesteatoma
Marsupialization of the sac or mastoidectomy.
Types of mastoidectomy
1. Cortical mastoidectomy i.e.removal of accessible mas-
toid air cells without interference with middle ear.
2 Radical mastoidectomy, where mastoid air cells, mastoid
antrum and middle ear cavity are debrided to form a
single smooth walled cavity accessible freely through
external auditory canal. Malleus, incus, chorda-
tympani and tympanic membrane are remove.
3. Modified radical mastoidectomy, where epitympanum,
mastoid antrum and external canal are converted to
a single cavity, but healthy ossicular and tympanic
membrane remnants are retained.
Types of tympanoplasty
Based on physiologic principles, there are six types of
tympanoplastic procedures:
Type I Myringoplasty only.
Type II Myringoplasty plus minor reconstruction of
ossicular.
Type III Myringoplasty plus reconstruction for mis-
sing malleus and incus.
Type IV Creation of hypotympanic air bubble.
92 Short Notes and Short Cases in ENT

Type V Fenestration of lateral semicircular canals


plus round window graft.
Type VI Sound inversion i.e., oval window is pro-
tected by graft and round window is
exposed to sound waves.
Pre-requisites for tympanoplasty
1. Adequate air bone gap with good cochlear reserve.
2. Mastoid free from disease.
3. Functioning eustachian.
Indications of myringotomy
1. Acute suppurative otitis media with bulging ear drum.
2. Persisting deafness after control of ASOM.
3. Secretory otitis media of eustachian catarh.
4. Adhesive otitis media.
5. Aero-otitis.
6. Meniere’s disease.
Short Cases 93

Case No. 6

A 40-year old female is complaining of gradually


increasing deafness without any aural discharge or pain
in the ears. Examination reveals conductive deafness
with normal mobility of tympanic membrane. Her
mother had a similar problem.

What is the probable diagnosis?


Otosclerosis
Is otosclerosis a misnomer?
Yes. The disease is not simple ankylosis of foot plate of
stapes. It is actually formation of new spongy bone,
better termed as “otospongiosis”.
Sites of predilection in otosclerosis
• Fossula ante fenestrum (Commonest site)
• Fossula post fenestrum
• Round window
• Foot plate of stapes
• Area below internal auditory meatus.
Clinical types of otosclerosis
• Stapedial otosclerosis
• Cochlear otosclerosis
• Mixed.
What is paracusis willis?
It means to hear better in noisy surroundings like in
public transport, machine shops and engine rooms. In
these places, normal person raises his voice, which is
above threshold of otosclerotic patient.
What is Gelle’s test?
In this test, the vibrating tuning fork is placed on
mastoid process and the pressure in the external
auditory canal is increased by Seigle’s speculum. In
normal persons, there is decrease in sound, but in
otosclerosis, there is no change as footplate of stapes is
already fixed.
Differential diagnosis of otosclerosis
• Otitis media
• Ossicular chain disruption
• Congential ossicular chain fixation
• Paget’s disease involving temporal bone.
94 Short Notes and Short Cases in ENT

Treatment of otosclerosis
• Sodium fluoride and calcium carbonate orally over
years may retard the progress of early disease.
• Hearing aid.
• Mobilization of fixed stapes, stapedectomy or
bypassing the stapes.
Contraindications for stapedectomy
• Otitis media
• Young children
• Poor cochlear reserve
• Cochlear otosclerosis
• Only functioning ear
• Mild disease
• Extensive tympanosclerosis.
What is cochlear otosclerosis?
It is the presence of otosclerotic lesions in the capsule
of cochlea, causing sensory neural deafness. It should
be suspected in any patient who develops bilateral
progressive sensory neural deafness in early adult life.
Malignant otosclerosis presents with rapidly pro-
gressive sensory neural deafness from childhood. As
there is no available surgery, hearing aid and fluoride
treatment are the only hope.
What is noise trauma?
Loud explosions can rupture tympanic membrane or
may cause haemotypanum. Spontaneous heating is
usual. When conductive hearing loss exceeding 30 dB
persists for more than 3 months, ossicular chain disrup-
tion be suspected demanding exploration and recon-
struction.
Common tumors of ear
External ear and external auditory canal
• Squamous and basal cell carcinoma
• Exotosis
• Osteoma
• Sebaceous adenoma and ceruminoma and adenocar-
cinoma.
Middle ear tumors
• Glomus jugular tumors.
• Embryonal rhabdomyosarcoma involving middle
ear.
• Eosinophilic granuloma involving temporal bone.
Short Cases 95

Inner ear
• Acoustic neuroma.
What is glomus jugular tumor?
Non-chromaffin paraganglionic tissue is present in the
dome of the jugular bulb or along the course of Jacobson’s
nerve or sometimes in mucosa of middle ear. Tumors
arising from this tissue are called glomus tumor,
chemodectoma, or non-chromaffin paraganglionoma.
Though histologically benign, it behaves as malignant
tumor.
Diagnosis of glomus jugular tumors
Symptoms Signs
Tinnitus (pulsatile) Red mass behind tympanic membrane or
Deafness red ploypoid bleeding mass in ear canal
Blood stained discharge Destruction of mastoid
7th and 8th cranial Carotid angiogram shows highly vascular
nerve palsy mass.
96 Short Notes and Short Cases in ENT

Case No.7

A young man has been complaining of intermittent


attacks of intense vertigo, tinnitus and deafness with
occasional vomiting. Examination during attacks reveals
mild sensoryneural deafness and impairment of
thermally induced nystagmus on affected side.

What is the probable diagnosis


Meniere’s syndrome.
Pathogenesis of Meniere’s syndrome
There is endolymphatic hydrops probably due to
diseases of endolymphatic sac, which is responsible for
endolymphatic filtration and excretion. Syphilis and
head trauma are two known precipitating factors.
What is glycerol test?
Glycerol test is a valuable diagnostic tool for hydrops.
By increasing blood viscosity it reduces hydrops,
improves hearing and ameliorates vertigo. Patient is
given 1.5 cc glycerol/kg. Body weight alongwith similar
quantity of saline to drink. 15db improvement on
puretone audiometry or speech discrimination ability
implies a positive test and makes 60% probability of
endolymphatic hydrops.
What is vertigo?
Vertigo is a subjective sensation of motion, when there
is no motion at all or exaggerated sense of motion in
response to a given bodily movement. Vertigo can be
peripheral or central in origin.
Characteristics of peripheral and central vertigo
The vertigo that results from peripheral vestibulopathy
is of sudden onset, severe and incapacitating with
tinnitus and deafness.
Peripheral lesion Central lesions
Meniere’s syndrome Brainstem tumor
Labyrinthitis Vascular disease
Vestibular neuronitis AV malformation
Trauma to temporal bone migraine, multiple
Positional vertigo sclerosis
Acoustic neuroma
Short Cases 97

In central lesions, there is sensory motor disturbance,


with cranial nerve palsy and cerebellar deficit, but
auditory function is generally spared.
What is positioning vertigo?
It is transitional vertigo following changes in head
position. Positional vertigo means vertigo occurring
when certain head positions are maintained and indi-
cates vertebro-basilar insufficiency or cervical spondy-
losis. There is latency period of several seconds follo-
wing head movement before vertigo appears and it sub-
sides within a minute. Constant repetition of positional
changes leads to habituation. In central lesions, there
is no latency period or habituation.
When to suspect acoustic neuroma?
Progressive sensory neural deafness accompanied by
vertigo, tinnitus and ataxia speak of acoustic neuroma
at cerebellopontine angle. Appearance of cerebellar signs
and Vth/VIIth cranial nerve palsy further strengthen the
diagnosis.
Clinical and laboratory examination of a patient of
vertigo
• Gait evalution and Romberg test.
• Observation of nystagmus and effects of visual
fixation.
• X-ray skull for assessment of auditory canal width.
• Audiologic evalution.
• Calorie testing, electronystagmography.
• CT scan, brain stem auditory evoked response.
• Fukuda test, where patient marches in place with
eyes closed when labyrinth in diseased patient
rotates towards diseased labyrinth.
Management of acute attack of Meniere’s disease
• Low salt diet
• Nicotinic acid, Vit. A and D supplement
• Diuretics
• Vasodilators
• Betahistidine and cinnarazine.
Surgical therapy of disabling Meniere’s disease
• Destruction of labyrinth by ultrasound, cryoprobe
• Labyrinthectomy by transmeatal or transmastoid
route
98 Short Notes and Short Cases in ENT

• Vestibular neurectomy
• Shunt surgery to decompress endolymphatic sac
• Cervical sympathectomy
• Myringotomy with grommet insertion.
Drugs causing vertigo
• Streptomycin, vancomycin
• Aminoglycosides
• Anticonvulsants
• Quinine and salicylates
• Diuretics
• Antiepileptics
• Cytotoxic agents.
Management of an acute vertiginous attack
• Bedrest
• Diazepam IV
• Antihistamines like meclizine, cyclizine, dimenhydri-
nate
• Control of nausea and vomiting
• Physical exercise to the point of producing nausea
and vertigo in order to achieve central compensatory
mechnisms
• Surgical therapy as for Meniere’s disease.
Characteristics of neural hearing loss
• Marked deterioration of speech discrimination out
of proportion to the decrease in pure tone threshold.
• Associated vertigo is chronic and debilitating.
• Associated nystagmus is vertical, non-fatigable, with-
out latency and unsuppressed by visual fixation.
Causes of tinnitus
• Wax in internal auditory canal
• Middle ear disease stimulating tympanicplexus
• Sensory neural hearing loss
• Post-stapedectomy
• Acoustic neuroma
• Anaemia
• Idiopathic.
Management of acute tinnitus
• Lidocaine 100 mg. IV for immediate relief
• Tocainide 200 mg. tid
• Carbamazepine 200 mg.tid
• Vasodilators (nylidrine)
Short Cases 99

• Surgical-like stellate ganglion block, cervical sympa-


thectomy, cochlear nerve section and chorda
tympani resection.
• Masking and hearing aid.
What is presbyacusis?
It is sensorineural deafness of old age. It is due to senile
degeneration or organ of Corti and nerve fibres. Deafness
is bilateral, symmetrical, commonly affecting high tones.
Causes of sudden deafness
Conductive Sensori neural
Traumatic tympanic Haemorrhage into labyrinth
Membrane perforation Fracture of temporal bone
Haemotympanum Viral infections (mumps,
measles)
Aero-otitis Idiopathic
Ossicular chain disruption

Management of sudden deafness without a cause


• Vasodilators like nylidrine, betahistidine, nicotinic
acid
• ACTH and steroids
• Low molecular weight dextran
• O2 + CO2 to induce cerebral vasodilation
• IV conray, O2 + helium
Tests to identify hysterical deafness
1. Stenger’s test, on the principle that patient is unable
to identify identical sounds, when produced at
different distances from ears.
2. Weber’s test for sound localization.
3. Lombard’s test with noise box placed in sound ear.
4. Loud voice test.
5. For seeing lid contraction, pupillary size change on
the side of loud sound.
6. Stethoscope test.
7. Masking noises give variable audiometric results.
8. Hypnosis with IV pentothal and audiometry.
9. Auditory evoked potential.
Difference between hearing- aid and cochlear implant
Hearing-aids are only sound amplifers on certain
frequencies, while cochlear implant is an electrical
stimulating appliance. Hearing-aid is only helpful, if
100 Short Notes and Short Cases in ENT

some hair cells are still surviving with intact cochleo-


cerebral transmission. Implants are for dead cochlea or
defective transmission. Stimulation can be of cochlear
or acoustic nerve. Sound perception as a whole is
possible with implants, but discrimination is not there.
Test to identify a cochlear disease from disease of
neural pathway (retrocochlear)
• Tone decay test
• Short increment sensitive index test
• Speech discrimination test
• Bikesy audiometry.
What is impedance audiometry?
In this test, air pressure in external ear canal is varied
from 200 mm. H2O to + 200 mm. H2O and middle ear
impedance is measured. It is helpful in diagnosing adhe-
sive/secretory otitis media, ossicular chain disruption.
What is the best for hearing in a baby?
dB is expressed as 10 log t1/t2 or 20log p1/p2 where t1
and p1 are intensity expressed in watts and dynes/cm2
respectively of the existing sound and t2 and p2 as
intensity of a references sound.
Short Cases 101

Case No.8

A child has presented with pain in the throat, difficulty


in swallowing and fever. Examination reveals enlarged
tender jugulo-digastric glands and red congested tonsils.

What is your diagnosis?


Acute tonsillitis
What are tonsils?
Tonsils are subepithelial lymphoid collections situated
in between faucial pillars. They help in phagocytosis of
bacteria and thus protect alimentary and respiratory
tract from bacterial invasion.
Causative agents to tonsillitis
• Group A betahaemolytic streptococci
• Staphylococcus aureus
• Haemophillius influenzae
• Viruses
• Corynebacterium haemolyticum
• Chlamydia
• Mycoplasma.
Pathological types of tonsillitis
• Parenchymatous
• Follicular
• Membranous.
Complications of tonsillitis
• Chronicity
• Peritonsillar abscess
• Parapharyngeal abscess
• Acute otitis media
• Rheumatic fever.
What is Peritonsillar abscess (Quinsy)?
When infection penetrates the tonsillar capsule and
involves the surrounding tissue, peritonsilliar cellulite
results, ending up in peritonsilliar abscess. There is
trimus, dysphagia, ‘hot potato’ coice and deviation of
uvula and soft palate with cervical lymphadenopathy.
Immdiate drainage followed by tonsillectomy 6-8 weeks
later is advisable.
102 Short Notes and Short Cases in ENT

Cause of trimus in quinsy


Pterygoid muscle spasm.
Indications of tonsillectomy (bilateral)
• Repeated attacks of tonsillitis in spite of drug
prophylaxis.
• Chronic cervical adenopathy with tonsillitis causing
frequent ill-health, stunted growth.
• Tonsil serving as a source for chronic streptococcal
dissemination or producing attacks of rheumatic
fever.
• When tonsils are responsible for carrier state of
streptococcus and C. diptherae.
• Quincy, tonsils causing sleep apnoea, COPD.
Indications for unilateral tonsillectomy
• Excison biopsy of tonsil to determine a possible
malignancy (lymphoma).
• As an approach to expose glossopharyngeal nerve
or enlarged styloid process (stylgia).
• In branchial fistula, to remove the complete tract.
Contraindications for tonsillectomy
• Ongoing epidemic of poliomyelitis
• Pregnancy
• Bleeding disorder.
Complications of tonsillectomy
Bleeding—primary, reactionary or secondary; damage
to internal jugular vein.
Complications of Quinsy
Extension of abscess to deep neck., retropharyngeal and
posterior mediastinal spaces.
What are adenoids?
Adenoids are nasopharyngeal tonsils around the
pharyngeal opening of eustachian tubes. They are the
seat of infection in children between 3-6 years of age and
disappear around puberty.
Clinical presentation of enlarged adenoids
• Snoring, mouth breathing, persistent nasal discharge
and sinusitis.
• Recurrent attacks of otalgia, CSOM and often
deafness.
• Nocturnal cough due to postnasal drip.
• Rhinolalia.
Short Cases 103

What is adenoid facies?


Dull look, pinched nostrils, open mouth, narrow
maxillary arch, retracted upper lip and protruding teeth.
Diagnosis of enlarged adenoids
• Posterior rhinoscopy.
• Palpation of nasopharynx-adenoids have a feel like
bag of worms.
• X-ray skull lateral for adenoid mass.
Complications of adenoidectomy
• Damage of eustachian tube opening and soft palate.
• Atlanto-axial joint subluxation.
• Anterior vertebral ligament laxity.
Tumors of nasopharynx
Benign Malignant
Juvenile angiofibroma squamous carcinoma
Hamartoma, dermoid adenocarcinoma
Lymphosarcoma
lymphoepithelioma

Characteristics of nasopharyngeal cancer


• Common to males of Chinese origin.
• Relatively young age of onset.
• Direct intracranial spread after destruction of
basiocciput and basisphenoid.
• Involvement of 5th, 6th, 9th, 10th, 11th, &12th cranial
nerves.
• Can spread anteriorly into nasal cavities, ethmoids,
orbits.
• Lymphatic spread to cause bilateral cervical adeno-
pathy or eustachian block with otorrhoea, conductive
deafness.
• Epistaxis.
• Metastasis to lungs and bones (particularly spines).
Treatments and prognosis of nasopharyngeal cancer
• Chemoradiotherapy is the primary treatment.
• Prognosis is poor; because of early intracrainal
spread and neck mode involvement.
Common tumor oropharynx and its treatment
The common benign tumors of oropharynx are papil-
loma and plemorphic salivary adenoma, which need
104 Short Notes and Short Cases in ENT

surgical excision. Malignant tumors are squamous cell


carcinoma and lymphosarcoma, which need radio-
therapy often preceded by excision.
What is lipoepithlioma?
This tumor is common to nasopharynx and oropharynx,
where there are subepithelial lymphoid tissue collec-
tions. It occurs in young people, with early and wide-
spread metarasis. Radiotherapy is the treatment of
choice.
Short Cases 105

Case No 9.

A 50-year old male has been complaining of hoarseness


of voice for fast 2 months. Examination reveals painless,
hard, nontender cervical lymphadenopathy.

What is the presumptive diagnosis?


Laryngeal malignancy is most probable.
Classification of malignant tumors of larynx
The malignant tumors may involve supraglottis, glottis,
and subglottis. Supraglottis cancers may be in aryepig-
lottic folds, arytenoids, infrahyoid epiglottis, ventricular
bands, and ventricular cavity. Glottis cancers may
involve vocal cords, anterior and posterior commisures.
Predisposing factors for laryngeal cancer
• Smoking
• Alcohol
• Tobacco
• Voice abuse.
Common site of laryngeal malignancy
Free margin of upper surface of true vocal cords in
anterior two-third.
Symptoms of laryngeal malignancy
• Hoarseness—common in glottis
• Cough—Supraglottic tumor
• Haemoptysis, Stridor odynophagia.
.
What is Stridor and its significance?
Stridor is a high-pitched sound produced by lesions that
narrow the airway. Airway narrowing above vocal cord
produces predominantly inspiratory Stridor, while that
below vocal cord produces either expiratory or mixed
stridor.
Implications of neck node enlargement in laryngeal
mlignancy
Lymphnode involvement in glottic cancer is rare as the
vocal cords are practically devoid of lymphatic. Supra-
glottic cancers have early lymph node metastasis, while
subglottic cancers metasize to pre- and paratrachial and
mediastinal nodes.
106 Short Notes and Short Cases in ENT

Indirect laryngoscopic appearance of laryngeal cancer


Nodular, ulcerative or papilliferous lesion with or
without fixation of vocal cords.
Cervical lymphnode biopsy findings in laryngeal
cancer
Metastatic squamous cell carcinoma.
Treatment of laryngeal cancer
• Glottic cancer—Radiotherapy.
• Subglottic cancer—Total laryngectomy followed by
radiotherapy for neck and superior mediastinal
nodes.
• Supraglottic cancer—Radiotherapy or surgery.
With radiotherapy, cure rate for early tumors is 85-95%.
Conservation surgery or total laryngectomy is necessary
for radiation failures and more advanced disease. Use
of tracheoesophageal valve following laryngectomy
restores useful speech.
Common benign tumors of larynx
• Papilloma
• Haemangioma
• Fibroma
• Chondroma
• Singer’s nodule
• Polypoid changes.
What is Singer’s nodule?
Singer’s nodules are smooth paired lesions from
junction of anterior one-third and posterior two-third of
vocal cords. In children, they are called screamer’s
nodules. They are due to vocal abuse and cause
hoarseness. Treatment requires modification of voice
habits and speech therapy. Surgery is reserved for
recalcitrant nodules.
What is the significance of polypoid changes in vocal
cords?
Polypoid changes in vocal cords may result from vocal
abuse, smoking, chemical industrial irritants and may
occur in hypothyroidism. Care of the above inciting
factors and steroid inhalation cause regression of
mucosal changes, else the hyperplastic vocal cord
mucosa is to be removed.
Short Cases 107

Mechanism of hoarseness in GI reflux


Patient of GI reflux often contact ulcers on vocal
processes of arytenoid cartilages. H2 receptor blockers
and foaming antacids alongwith elevation of head-end
of bed are all that required.
What is intubation hoarseness
Tracheal intubation during anesthesia may be followed
by granuloma formation posteriorly between the vocal
processes, to cause hoarseness.
108 Short Notes and Short Cases in ENT

Case No.10

A 30-year old, who suffered from throat pain, fever,


coryza and dry cough two months ago is complaining
to have developed hoarseness and occasional dry cough
thereafter without any constitutional symptoms. Indirect
laryngoscopy shows hyperemic thick vocal cords, with
normal movements and some oedema of laryngeal
mucosa.

What do you think of this case?


A case of chronic simple laryngitis.
What is croup?
Croup is a viral infection of subglotic region and the
tracheobronchial tree. It presents with barking cough
and stridor, both inspiratory and expiratory. Dexame-
thasone, either IM/IV shortens the duration of ailment.
How epiglottitis differs from Croup?
• Laryngomalacia
• Croup, laryngeal foreign body
• Laryngeal diphtheria
• Laryngismus stridulous
• Vocal cord paralysis
• Papilloma of larynx
• Laryngeal webs, cysts and bifid epiglottis.
What is laryngomalacia?
Laryngomalacia is the common cause of Stridor infancy.
It is due to immature omega-shaped epiglottis, that
prolapes into the airway during inspiration. There is also
flabbiness of aryepiglotic folds. Voice is normal. Stridor
tends to be worse with child lying supine. It gradually
disappears by the age of 2-3 years.
When to suspect laryngeal diphtheria
A toxic febrile patient, with cervical lymphadenitis (bull
neck) showing greyish yellow membrane over tonsils,
soft palate and posterior pharyngeal wall and has
marked dyspnoea and stridor.
What is Reinke’s oedema?
It is bilateral polypoidal degeneration of membranous
vocal cords, producing smooth oedematous swelling of
Reinke’s space. Indirect laryngoscopy shows bilateral
Short Cases 109

pale spindle-shaped swellings of the vocal cords,


commonly arising from vocal abuse. Treatment is by
excision of the strip of mucosa from membranous cords.
Predisposing factors for chronic laryngitis
• Voice abuse
• Smoking
• Infection in adenoids, tonsils and sinuses
• Air pollution.
Consequence of chronic laryngitis
Hyperkeratosis and leukoplakia may follow chronic
laryngitis. Leukoplakia looks like a white patch or
nodular plaque on vocal cord, while hyperkeratosis
appears as having warty or greyish papillomatous
appearance. Both need microsurgical excision.
What is the nature of laryngeal involvement in
tuberculosis?
The posterior part of larynx is commonly affected, inclu-
ding laryngeal ventricles, false cords and arytenoids.
The initial stage of infiltration is followed by a
proliferative phase, when tumors like masses appear.
The ulceration occurs, finally healing by fibrosis.
Symptoms like painful deglutition (odynophagia),
weakness and hoarseness of voice are added on
symptoms of pulmonary tuberculosis. Adduction
weakness of vocal cords is an early sign. Ulcers with
undermined edge, turban epiglottis and mucosal
oedema are evident in direct laryngoscopy.
What are vocal cords?
Vocal cords are fibroelastic bands that extend from angle
of thyroid cartilage anteriorly to vocal process of aryte-
noids posteriorly. They are formed by reflection of
mucosa over vocal ligaments, which are free edges of
cricovocal membrane. The vocal cords have stratified
squamous epithelium with no submucous layer and no
lymphatics.
Nerve supply of larynx
Superior laryngeal nerve is sensory to laryngeal mucosa
above vocal cords and is the motor nerve for cricothyroid
muscle. Mucosa of larynx below the vocal cords and all
other intrinsic muscles are supplied by recurrent
laryngeal nerves.
110 Short Notes and Short Cases in ENT

Causes of hoarseness of voice


• Recurrent laryngeal nerve palsy
• Laryngeal tumors; bengin and malignant
• Laryngitis
• Vocal cord granuloma, polyp, oedema, fixation.
Causes of unilateral vocal cord palsy
• Thyroid surgery
• Pancoast’s tumor
• Mediastinal adenopathy, aortic anecurysm, mitral
stenosis
• Jugular foreman syndrome
• Cranial neuropathy.
Treatment of unilateral vocal cord palsy
• Medialization laryngoplasty
• Thyroplasty
• Teflon injection into paralysed cord.
Features of bilateral vocal cord palsy
• Stridor is the main symptom if palsy is of sudden
onset.
• If of insiduous onset, symptoms are minimal.
• Vocal cords are immobile. Voice may be retained as
the cords are apposed in midline.
• Thyroid surgery, neck trauma and tumor invasion
from anapestic thyroid or oesophageal malignancy
are responsible.
• Cricoarytenoid arthritis as in advanced rheumatoid
arthritis may cause fixation of vocal cords and be
excluded.

Potrebbero piacerti anche